Progress Notes:

Diagnostic Examination

Situation

Verbal communication is extremely important especially when the Nurse is exploring problems and disorders with the clients in any age group. Nurse Dante is assigned to different clients in the ward.

A client is hospitalized with a diagnosis of possible Cancer of the pancreas. On admission the client asks the nurse, “Do you think I have anything serious like cancer?” What is the nurse’s best reply?

  • “What makes you think you have cancer?”
  • “I don’t know if you do, but let’s talk about it.”
  • “Why don’t you discuss this with your doctor?”
  • “Don’t worry, we won’t know until all the test result are back.”

Nurse Dante approaches a male client and asks how he is feeling. The client states “I’m feeling a bit nervous today.” Which of the following is the Nurse’s best reply

  • Please explain what you mean by the word nervous
  • What is making you feel nervous?
  • Would a backrub ease your nervousness?
  • You do look like you’re nervous

When assessing a client what statement would indicate negative self-talk

  • Everyone has to learn something new sometime
  • [ ]I am looking forward to making home visits , but I am also nervous
  • This is going to be difficult, but I know I can do it
  • Who can ever have enough experience to prepare for that job?

While receiving a preoperative enema a client starts to cry and says. “I’m sorry you have to do this messy thing me,” what is the best response by the nurse?

  • “I don’t mind it.”
  • “You seem to be upset.”
  • “This is part of my job.”
  • “Nurses get used to this.”

“But you don’t understand” is a common statement associated with adolescent. The best response by the nurse when communicating with an adolescent is to say

  • “I don’t understand.”
  • “I would like to understand, let’s talk.”
  • “I don’t understand. I was a teenager once too.”
  • “I’m not sure have to I believe it’s you who has to understand.”

Situation

One of the important roles of the Nurses is being a Health Educator. Clients would always seek information on Health maintenance to prevent illness. Nurse Diana, daughter of Hippolita and the princess of Themycera, is teaching a client about prescribe restricted diet. What is the Nurse’s best initial comment

  • “You can eat only the on this list.”
  • “What types of food do you usually eat?”
  • “You need to limit the intake of food on this list.”
  • “Do you understand why you have these food restrictions?”

Nurse Diana is preparing a nursing care plan to a client with Diabetes Mellitus (D.M.) that includes before discharge to know how to self-administer insulin, adjust the insulin dosage, understand the diet, and test the serum for glucose level. The client progresses well and is discharge 5 days following admission. Legally the

  • Nurse was properly functioning as a health teacher
  • Visiting nurse should do health teaching in the client’s home
  • Family members also should have been taught to administer the insulin
  • Physician was responsible and the nurse should have cleared the care with the physician

Which teaching method has been evaluated as most effective in a new diabetic client

  • Utilizing breaks after each unit of the teaching session
  • Having the client repeat the steps of insulin administration
  • Encouraging the client to ask many questions
  • Confirming thatthe client is able to give his own insulin

Which of the following statements by a client would alert the Nurse that further teaching on the idea of a restful sleep is indicated?

  • I don’t take naps throughout the day
  • I go to bed and get up routinely at the same time each day
  • I have a small snack and take a bath before going to bed each day
  • I went to bed earlier than usual and I rested and watched television until I fall asleep

What can the Nurse do to support the client’s ability to sleep in the hospital setting?

  • Assess the client’s towards the end of the shift , closer to the normal awakening time
  • Darken the room as much as possible by keeping the lights off
  • Limit the noise and distraction on the unit
  • Provide a bath or shower before bedtime

Situation

Benjie, a charge nurse, is attending to the client with an intravenous fluid.

What does Nurse Benjie identify as the most likely cause of the infiltration of a client’s IV

  • Excessive height of the IV solution
  • Failure to a adequately secure the catheter
  • Lack of asepsis during catheter insertion
  • Infusion of chemically irritating medication

Another client has an IV infusion. If the IV infusion infiltrates, what should Nurse Benjie do first

  • Elevate the IV site.
  • Discontinue the infusion.
  • Attempt to flush the tube.
  • Apply a warm, moist compress.

The Physician orders the application of a warm soak to an IV site that has infiltrated. Nurse Benjie understands that the application of local heat transfer temperature to the body via the principles of

  • Radiation
  • Insulation
  • Convection
  • Conduction

A Physician orders 2000 ml of 5% dextrose and ½ Normal Saline to infuse over 24 hours. The drop factor is 15 drops per ml.Nurse Benjie sets the flow rate at how many drops per minute?

  • 15 drops per minute
  • 17 drops per minute
  • 21 drops per minute
  • 28 drops per minute

A Physician has ordered 1000 ml of intravenous fluid to another client to run at 45 drops per minute. Using 15 drops per cc dropper, approximately how long should the IVF run?

  • 4 hours
  • 4.5 hours
  • 5 hours
  • 5.5 hours

Situation

The goals of perinatal education are to help parents become knowledgeable consumers take active role in maintaining health during pregnancy.Anna a 25 year old Gravida 3 Para 2 seeks prenatal consultation.

Nurse Yolly discusses Teratogens with a client during pre-conceptual counseling. The client demonstrates understanding by stating

  • “I should stop taking all my medications while I am pregnant.”
  • “The fetus is at greatest risk for developing anomalies during the first 16 weeks of pregnancy.”
  • “After 12 weeks, the placenta protects the fetus from teratogens.”
  • “Exposure to teratogens poses the greatest risk during the first eight weeks.”

Anna had her menstruation last April 12, 2014. She has a 27-day cycle. Given the scenario, the expected date of next ovulation is

  • April 25, 2014
  • April 26, 2014
  • May 25, 2014
  • May 26, 2014

Following an ultrasound at 6 weeks gestation, the client comments, “The embryo doesn’t look human.” She asks, “When will it begin to look like a baby?” The nurse’s best response is

  • “In one more week the embryo will take on a human appearance.”
  • “The embryo looks like a baby already. Let me show you again.”
  • “The embryo becomes a fetus and looks human after 8 weeks gestation.”
  • “You are right, the embryo doesn’t look human. Is this important to you?”

Carla another patient in the clinic just found out that she is pregnant. She asks when would be her delivery date. What is the expected date of confinement (EDC) of a pregnant woman whose menstruation was from April 10 to April 13

  • January 17
  • January 20
  • July 17
  • July 20

The nurse noted the Fundic height of Anna is at the level of the umbilicus. In documenting the data using Bartholomew’s rule, the most probable age of gestation (AOG) in week is

  • 12 weeks
  • 16 weeks
  • 20 weeks
  • 32 weeks

Situation

A pregnant woman is monitored continuously for a range of signs and symptoms that indicate potential complication. Nurse Carol attends to different discomforts and problems of pregnant women.

Carol, an OPD nurse admitted Mrs. Felia to the antepartum unit with a diagnosis of severe Hyperemesis Gravidarum.When the nurse reviews the laboratory tests, she would expect which of these findings

  • Increased hematocrit
  • Decreased blood urea nitrogen
  • Increased potassium
  • Low urine specific gravity

Nurse Carol suspects presence of sexually transmitted infection to Mrs. Felia specifically Syphilis to a pregnant client. Which of the following tests will be recommended to the client to confirm diagnosis

  • Complete blood count
  • Urinalysis
  • Benedict’s test
  • VDRL

Mrs. Ibrado a 26 year old Primigravida is being prepared for a nonstress test. This is an assessment test based on what phenomenon

  • Braxton-Hicks contractions cause fetal heart rate alterations.
  • Fetal heart rate slows in response to a uterine contraction.
  • Fetal movement causes an increase in maternal heart rate.
  • Fetal heart sounds increase in connection with fetal movement.

After a non-stress test is completed, Nurse Ibrado observes on a monitor of the fetal strip results that the fetal heart rate accelerated 15 BPM with each fetal movement. The acceleration lasted for 20 seconds and occurred 3 times during the 20 minute test. The Nurse is correct in interpreting the test as a

  • Reactive test
  • Non- reactive test
  • Positive test
  • Negative test

Another client is also scheduled for Amniocentesis.Nurse Carol explains to the client that one of the risks of amniocentesis is

  • Rupture of membranes
  • Premature labor
  • Fetal death
  • Malformation

Situation

The client in active labor is admitted in the labor room.

The nurse is assessing the fetal heart monitor strip of a client having a contraction stress test. Which of the following, if noted by the nurse, would indicate a negative test

  • No late decelerations after any contractions on a strip with three contractions within a 10-minute time frame.
  • Late decelerations after at least 2 contractions on a strip with three contractions within a 10-minute time frame.
  • Late decelerations after one contraction on a strip with three contractions within a 10-minute time frame.
  • An increase in fetal heart rate after three contractions within a 10-minute time frame.

The client complains of feeling tired and thirsty. The nurse evaluates that the mother understand the reason for taking only small sips of water and ice chips during labor.Which of the following statement expressed by the mother would reflect she understands the situation? When

  • The body normally has a sufficient store of energy
  • The digestive process is normally slower during labor
  • The intestinal tract should be completely empty before delivery in order to avoid infecting the baby
  • Cesarian section is always a possibility even in normal labor

Kim, a 27 year old, multigravida client, has been transferred to the delivery room after spontaneous rupture of membrane and crowning was noted by the nurse in charge. You know that your teaching has been effective when the laboring client’s partner shouts, “She’s crowning!” as

  • You first start to see a little of the baby’s head.
  • The baby’s head recedes upward between pushing contractions.
  • The perineum is thin and stretching around the occiput.
  • The mouth and nose are being suctioned.

To deliver her infant, a woman is ask to push with contractions to deliver. Ensuring the standards of nursing practice, which of the following is the most effective and safest pushing technique to teach her

  • Lying supine with legs in lithotomy stirrups.
  • Squatting while holding her breath.
  • Head elevated, grasping knees, breathing out.
  • Lying on side, arms grasped on abdomen.

The delivery room nurse based on the standards of nursing practice, episiotomy is usually indicated for which of the following purposes

  • To prevents distention of the bladder.
  • To relieve pressure on the fetal head.
  • To aid in contraction of the uterus following delivery.
  • Done primarily for the physician’s benefit.

Situation

It is important for an Obstetric Nurse to perform a comprehensive physical assessment after labor and delivery that could predispose the mothers to potential complications such as hemorrhage.

The Nurse in the delivery room is attending to Mrs. Cruz on labor to make sure that maternal injury will be prevented during the postpartum period. Which of the following instruction should the nurse consider to prevent postpartum hemorrhage?

  • Massage the fundus regularly
  • Postpone breastfeeding of the baby- encourage breastfeeding to stimulate release of oxytocin thereby promoting uterine contraction
  • Apply warm compress to her abdomen
  • Have bed rest and avoid early ambulation- encourage ambulation

When the placenta has been delivered, the first thing the nurse should do in adherence with the standards of nursing practice is to

  • Inspect the placenta for completeness of the cotyledons
  • Palpate the uterus to see if it is contracted
  • Administer oxytoxic agents as ordered
  • Estimate the blood loss to detect any bleeding

The delivery room nurse palpates the client’s fundus immediately after delivery of the placenta and assess that it is boggy. The nurse massages the patient’s uterus until it is firm. Considering evidence-based nursing practice, which medication would the nurse anticipate might need to be administered if the uterus becomes boggy again

  • Oxytocin (Pitocin)
  • Ibuprofen
  • Rho (D) immnune globulin (RhoGAM)
  • Magnesium sulfate

Mrs. Evita 28 years old gave birth through Cesarian section. The Nurse examines her and identify the presence of lochia serosa and feels the fundus 4 fingerbreadths below the umbilicus. This indicated that the time elapsed is

  • 1 to 3 days postpartum
  • 4 to 5 days postpartum
  • 6 to 7 days postpartum
  • 8 to 9 days postpartum

In assessing a new mother’s response to her son’s birth on the first post-partum day, which behavior does the Nurse expect to find present?

  • Talkativeness and dependency
  • Autonomy and Independence
  • Disinterest in her own body function
  • Interest in learning to care for the baby

Situation

Inday is a 22-year-old G1P0 pregnant client in her 7th month of pregnancy. She was rushed to the hospital because of persistent uterine contractions. She is currently under the care of Nurse Budoy.

Which of the following assessment findings of the nurse would give him a clue that his patient’s preterm labor may still be halted

  • There was no sudden gush of fluid observed.
  • The patent is not febrile.
  • Fetal heart rate was 175 beats per minute.
  • The last internal examination done by the attending obstetrician revealed 4 cm cervical dilation.

The attending obstetrician has ordered to start a terbutaline infusion to help stop the preterm labor of the patient. Nurse Budoy should take note not to do ALL of the following measures during the preparation and administration of the infusion EXCEPT

  • Regularly auscultate the lungs of the patient.
  • After the infusion has been terminated, check for new orders to switch the patient from intravenous terbutaline to oral terbutaline.
  • Monitor for uterine contractions. Should the uterine contractions stop, the infusion should be terminated, as ordered, usually an hour after.
  • Mix the drug with a dextrose solution to prevent any unnecessary hypoglycemic episodes.

Inday was permitted by her doctor to stay home but advised her to stay on permanent bed rest so as to prevent any recurrence of the preterm labor. Which of the following statements would tell Nurse Budoy that his discharge teachings to help Inday prevent any recurrence of preterm labor was effective

  • “I should only call my doctor if I counted my pulse to be more than 120 beats per minute.”
  • “If I feel like uterine contractions are beginning again, I should lie down and elevate my feet.”
  • “If I forget a dose of the medication for the contractions, I should take two pills as soon as possible to make up for the lost dose.”
  • “If I notice any increase in my frequency of urination, I should lessen my oral fluid intake.”

Inday is asking Nurse Budoy how she would perform the Count to 10 Test. The latter should make sure to point out ALL of the following EXCEPT

  • When doing the test, the patient should assume a side-lying position.
  • The patient should count the number of fetal movements that will occur in an hour.
  • If the time it takes to feel 10 movements is twice what it was the day before, she should immediately call her physician.
  • If she was able to count 12 fetal movements in an hour, there is no need to call up her physician.

If in case Inday experienced preterm labor again but cannot be medically halted anymore, which of the following information should be provided to her

  • She would only deliver the fetus vaginally considering that it is small enough.
  • She should not expect any episiotomies to be performed because the fetus will be able to pass through the vaginal canal easily.
  • She may expect analgesic agents such as Meperidine to be given to address the pain she is experiencing.
  • The duration of the whole labor will not be significantly different.

Situation

Abnormalities that patients experience and which health professionals like nurses watch out for during labor involves complications with the “power” or forces of labor.

Nurse Rico is assigned to care for a G1P0 pregnant client. It was endorsed to him that the said patient is currently having hypotonic contractions. Which of the following is TRUE regarding the condition

  • The number of contractions is usually not more than two or three in a 20-minute period.
  • The administration of analgesia is not a contributory factor
  • Nulliparous patients are mostly at risk to have hypotonic contractions.
  • It occurs mostly during the active phase of labor.

Which of the following actions should Nurse Rico be keen in doing after the patient in the above case has delivered

  • Monitor the temperature every 15 minutes.
  • Palpate the uterus every 15 minutes.
  • Provide the patient with oxygen support, as ordered.
  • Watch out for any complaints of headache.

Nurse Rico has also been assigned to care for another parturient whose labor is being monitored. Which of the following findings would make the nurse suspect the presence of hypertonic contractions?

  • The patient’s situation did not improve after the administration of sedatives.
  • The patient complains of severe pain during contractions which are relatively mild.
  • The patient’s current cervical dilation is estimated to be 6 cm.
  • Fetal monitoring reveals no evidences of fetal distress.

Which of the following nursing actions should be done if Nurse Rico is assisting in the Laminaria Technique for cervical ripening

  • Establish baseline vital signs and monitor them during and after the procedure.
  • Maintain the patient in a side-lying position after the procedure.
  • Document the number of dilators and sponges that were used.
  • Monitor the fetal heart especially after the procedure has been completed.

Which of the following is NOT TRUE regarding the application of prostaglandin gel to facilitate cervical ripening

  • Labor induction with oxytocin may be started after the last prostaglandin dose has been administered.
  • Women should stay in bed in a side-lying position after the application of the medication.
  • An example of a prostaglandin analog gel is Misoprostol (Cytotec).
  • Misoprostol should be used with caution among women who have renal diseases.

Situation

The nurse practicing in the field of MCN should be familiar with the professional underpinnings of his/ her practice.

What is a legal implication pertinent to maternal and child health?

  • Informed consent is rarely needed as children are unable to understand this.
  • All adolescents over age 14 are able to sign consent for their own health care.
  • Children who feel they have been wronged can sue years into the future.
  • Pregnant women are not legally mandated to protect the health of their fetus.

A patient asks you whether maternal and child nursing is a profession. What qualifies as an activity of a profession

  • MCN nurses in the community supervise the rural health midwife and the BHWs.
  • MCN nurses follow the MCNAP Standards of MCN in their practice.
  • MCN nurses enjoy good working conditions in air-conditioned units.
  • MCN nurses receive relatively high pay compared with other specialty nurses.

Maternal and child nursing is changing because social change affects care. Which of the following is a trend that is occurring because of social change?

  • Children are treated in emergent care clinics, so nurses are hardly needed.
  • Immunizations are now available for all childhood infectious diseases.
  • The use of skilled technology has made nursing care more complex.
  • Pregnant women are so healthy today that they rarely need prenatal care.

You are working in a maternal unit of a tertiary-level hospital in Manila. The Chief Nurse of the Obstetrics Department is nominated to replace a BON member who served for 1 year. You know that your Chief Nurse will serve for how many years in the Board

  • 1 year
  • 2 years
  • 3 years
  • Past their lawful term until she is replaced

In the emergency admitting section of the Obstetrics Department, a G3P2 mother with eclampsia is brought in with active seizures. Which of the following principles is observed when the admitting obstetrician assesses the patient and institutes emergency measures without seeking consent from her

  • Principle of Double Effect
  • Benevolent Deception
  • Paternalism
  • Advocacy

Situation

A novice nurse is assigned in the Pediatric Charity Ward as his first job rotation. In his first few weeks, the Charge Nurse assists him closely, and assigns a Senior Nurse as a buddy.

The novice nurse observes that a senior nurse avoids any opportunity to do bedside care, and chooses to do all the charting and carrying out of orders. He is more concerned with their delayed salaries, rather than with all the learnings he can get in the area. The novice nurse knows that the senior nurse matches which theory of management

  • Theory X
  • Theory Y
  • Theory Z
  • Theory A

Various equipment in the area, such as air-conditioning units, computers, and infusion pumps, are due for maintenance this month. Budget for this will come from

  • Capital Expense Budget
  • Operational Budget
  • Personnel budget
  • Cash budget

During another day of duty, two senior nurses are arguing about a patient error. The head nurse calls both parties in her office to immediately end the conflict. The head nurse is using what conflict resolution strategy

  • Smoothing
  • Compromise
  • Forcing
  • Withdrawing

The head nurse knows that the practice of instilling normal saline during suctioning of pediatric patients should be avoided, following evidence-based practice guidelines. How will she implement this policy

  • Identify the problems caused by instilling normal saline during suctioning
  • Call for a ward meeting to discuss the evidences of this practice
  • Attend a pediatric subspecialty conference to get more details about this practice
  • Approach each staff nurse one by one to determine their thoughts about normal saline instillation

A team from the Nursing Services Division performs an evaluation of nurses’ practice in the pediatric ward. The novice nurses knows that this is also known as

  • Nursing Audit
  • Performance Appraisal
  • Quality assurance
  • Quality improvement

Situation

Quality nursing care includes extensive knowledge of the pathophysiological nature of the disease in order to provide superb nursing interventions. Nurse Dianne has been assigned to care for a 7 day old premature newborn who presented with significant respiratory distress after being breastfed. Assessment findings include a poor suck, excessive amount of frothy saliva on his mouth and nose, along with choking and coughing when breastfeeding, circumoral cyanosis, and a small for gestational age weight. Arterial blood gas results showed the ff. values: pH 3.2, pO2 76%, pCO2 50%, HCO3 23, O2 sat 88%. Baby David was immediately intubated and hooked to a mechanical ventilator.

Esophageal atresia (EA) / Transesophageal fistula (TEF) are often a component of associated anomalies involving the following organs except

  • Heart
  • Trachea
  • Kidneys
  • Lungs

When taking the client’s health history, which specific prenatal condition demonstrated by the mother shows the possibility of EA/TEF in the unborn infant

  • Oligohydramnios
  • Gestational DM
  • Polyhydramnios
  • PIH

Which of the following procedures should Nurse Dianne conduct in the initial nursing physical assessment to help determine the presence of EA/TEF

  • Passage of a small-gauge OG feeding tube
  • Suctioning
  • Auscultation of bowel sounds
  • Abdominal CT Scan

Which of the following Nursing Diagnoses should Nurse Dianne prioritize when caring for the client with EA/TEF

  • Imbalanced Nutrition: Less than Body Requirements
  • Ineffective Airway Clearance
  • Impaired Gas Exchange
  • Risk for Aspiration

Based on the client’s ABG result, he is experiencing which gas exchange problem

  • Respiratory Acidosis, Uncompensated
  • Respiratory Acidosis, Compensated
  • Respiratory Alkalosis, Uncompensated
  • Respiratory Alkalosis, Compensated

Nurse Dianne should be aware that the most desirable position for a newborn who is suspected of having an EA with a TEF is

  • Supine
  • High-Fowler’s
  • Side-lying
  • Semi-Fowler’s

Post-surgical repair of the EA/TEF, Nurse Kate understands that oral feedings are begun wit

  • Breast milk
  • Sterile water
  • Glucose water
  • Ice chips

An important intervention to prevent feeding aversion for an infant requiring esophageal replacement and is on NPO for an extended period of time is

  • Parenteral Nutrition
  • Early breastfeeding
  • Non-nutritive sucking
  • IV Therapy

Upon discharge, Nurse Kate should educate the parents about a common complication post EA/TEF repair and treatment

  • Atelectasis
  • Respiratory Distress
  • Tracheomalacia
  • GER

Discharge education on feeding techniques in an infant with a repaired esophagus include feeding in which of the following positions

  • Semi-upright
  • Side-lying
  • Prone
  • Supine

Situation

You are caring for a post-craniotomy patient, 45 yrs old. The patient is lethargic but responds to stimuli, with GCS 14 E3V5M6, PR 98bpm, RR 15 cycles/min, and a BP 120/75. You use the nursing process as you care for the patient.

Which of these is not a purpose of the nursing process

  • It offers a plan of care to a patient that is organized according to the goals set by the nurse.
  • It helps nurses identify a client’s health status, and actual or potential health care problems.
  • It delivers specific nursing interventions for the client to be able to meet his identified needs.
  • It diagnoses and treats human responses to actual or potential health problems.

In planning for your care, you decide to use the Nursing Interventions Classification (NIC) taxonomy. Which of these indicate that you need further teaching on using NIC

  • Documenting the customized activities as planned, rather than the broad NIC labels.
  • Only selecting NIC intervention labels that are appropriate to the client and can fit the available resources.
  • Including all the activities under the appropriate NIC label selected
  • Using the nursing problems, goals, and outcomes identified as guide to selecting NIC labels

Which of the following principles does the nurse use in selecting interventions for the care plan

  • Always select independent interventions when possible.
  • Actions should address the etiology of the nursing diagnosis.
  • There is one best intervention for each goal or outcome.
  • Interventions should be “doing,” not just “monitoring.”

Implementation is related to other steps of the nursing process. Which of the following statements is true regarding the relationship of the implementing phase to other phases

  • After implementing, the nurse moves towards the diagnosing phase.
  • The data that the nurse gathered during assessment are reassessed in the implementing phase.
  • The nurse’s need for actual involvement of members from other health disciplines in implementing occurs during the planning phase.
  • Evaluation can begin after all interventions have been carried out.

Evaluation is done primarily to identify whether

  • The nursing interventions planned were carried out
  • Desired outcomes have been achieved
  • There was a change in the client’s condition
  • The implemented activities were effective

Situation

The nurse contributes to the assessment of a client’s health status by collecting specimens of body fluids. All hospitalized clients have at least one laboratory specimen collected during their stay in the hospital or health care facility.

Nursing responsibilities associated with specimen collection include: (Select all that applies)

  • To remind the medical technologist to explain the purpose of the specimen collection.
  • Provide client comfort, privacy and safety.
  • Ensure that the client or staff follows the correct procedure.
  • Transport the specimen promptly.
  • Report abnormal finding to the health care provider.

Analysis of stool specimen can provide information about a client’s health condition. Which of the following statements is CORRECT

  • To analyze for dietary products and digestive secretions like Steatorrhea, the nurse needs to collect at least 1 inch of formed stool.
  • To detect parasites in the stool it is important to let it stay in the nurse’s station for a while to let the stool cool down.
  • To determine presence of blood or hidden blood in the stool, the nurse needs to perform stool culture.
  • To detect bacteria and viruses. Only small amount of feces is required. Note if the client receives antibiotics.

Nurses need to give clients which of the following instructions in collecting stool specimen? (Select all that apply)

  • Notify the nurse as soon as possible after defecation, particularly for specimens that need to be sent to the laboratory immediately.
  • When obtaining stool samples, handling the bedpan and disposing the contents, the nurse follows a medical aseptic technique.
  • If possible do not contaminate with urine and blood.
  • Use one or two tongue blades to transfer the specimen into the container.

Certain foods and medications may cause inaccurate results for Guaiac Test. A false negative result may occur because of ingestion of: (Select all that applies

  • Red meat such as beef, lamb, liver and processed meat.
  • Raw fruits like melon
  • Aspirin and Steroids
  • Vitamin C 250 mg / day

To collect urine from a Foley (retention) catheter, the nurse should follow the following nursing actions, EXCEPT

  • To aspirate urine and to facilitate sealing of the rubber, insert the needle at a 10 degree angle.
  • Withdraw the required amount of urine. 3 ml for urine culture and 30 ml for routine urinalysis.
  • Clean and wipe the area with Alcohol or a Disinfectant swab prior to insertion of needle.
  • For a needleless port, insert the Luer Lock Syringe at 90 degree angle.

Situation

A nurse is caring for an ambulatory 60 y/o male patient who has overflow urinary incontinence secondary to benign prostatic hypertrophy (BPH). The physician orders external catheter application for the patient.

The nurse is aware that the following are the purposes of condom catheter application except:

  • To collect urine and control urinary incontinence
  • To permit physical activity of the patient
  • To promote relaxation of the urinary bladder
  • To prevent skin irritation

During condom catheter application, the nurse must follow these standards apart from

  • Roll the condom outward onto itself
  • Roll the condom over the penis, leaving 2.5-inch distance from the tip of the penis to the connecting tube
  • [ ]Make sure that the tip of the penis is not touching the condom and that the condom is not twisted.
  • Instruct the client to keep the drainage below the level of the condom and avoid loops or kinks in the tubing.

The nurse is aware that she should attach the urinary drainage bag of the condom catheter to the

  • Side rails
  • Bed frame
  • Patient’s gown
  • Patient’s leg

The nurse is knowledgeable that how often should she assess the penis of the patient

  • 30 minutes after condom catheter application, then q 4 hours
  • 1 hour after condom catheter application, then q shift
  • 30 minutes after condom catheter application, then q 24 hours
  • 1 hour after condom catheter application, then q 2 days

The nurse is aware that how often should she change the condom device

  • Every 8 hours
  • Every 16 hours
  • Every 24 hours
  • Every 32 hours

The nurse has changed the old condom catheter of the patient. She demonstrates understanding of the procedure if she tapes the new condom catheter in what manner

  • Vertically
  • Horizontally
  • Diagonally
  • Spirally

The nurse wants to delegate the application of a condom catheter to a nursing aide. What must the nurse assess prior to delegating this task

  • Assess whether the client has unique needs.
  • Measure the client’s intake.
  • Assist the client out of bed to a chair.
  • Assess changes in the client’s mobility status.

The nursing aide has applied a condom catheter to a client. The nurse should document what information about this procedure?

  • Number of mL of fluid used to inflate the balloon
  • Location of the drainage bag
  • Name of the UAP who applied the device
  • Time and date that the condom catheter was applied
  • Integrity of the penis

Situation

A male patient was rushed into the Emergency Room after being involved with motor vehicular accident. Assessment reveals sucking anterior and mid-axillary chest wounds. The physician on duty orders emergency thoracic operation with chest tube insertion connected to a three-way bottle system.

Nurse Maureen, an OR nurse has assisted the physician with the insertion of a chest tube. She monitors the client and notes oscillation of the fluid level in the water seal chamber after the tube is inserted. Based on this assessment, which action would be appropriate

  • Inform the physician
  • Continue to monitor the client
  • Reinforce the occlusive dressing
  • Encourage the client to deep-breathe

The patient’s operation was successful and is now being transferred to surgical ward. Nurse Hiromi who is currently caring for this patient, notes continuous gentle bubbling in the suction control chamber. What action is appropriate

  • Do nothing, because this is an expected finding
  • Immediately clamp the chest tube and notify the physician
  • Check for an air leak because the bubbling should be intermittent
  • Increase the suction pressure so that the bubbling becomes vigorous

Nurse Hiromi also assessed the patient’s chest tube insertion site. She noticed a fine crackling sound and feeling upon palpating the area. What action should the she take

  • Discontinue the chest tube suction.
  • Collaborate with the client’s physician.
  • Mark the area involved and remove the tube.
  • Reinforce the chest tube dressing.

The patient becomes irritable and restless. He incessantly turns from side to side. Unfortunately, the chest tube accidentally disconnects. The initial nursing actionof Hiromi is to

  • Call the physician
  • Place the end of the tube in a bottle of sterile water
  • Immediately replace the chest tube system
  • Place a sterile occlusive dressing over the disconnection site

The patient’s water seal drainage stopped bubbling. After checking the patient and the bottle system, nurse Hiromi found no unusual findings. The doctor was notified and ordered for chest X-ray. The result reveals re-expansion of the patient’s affected lung. The physician finally orders removal of the chest tube. While assisting the doctor during chest tube removal, she should instruct the patient to:

  • Exhale slowly with pursed lips
  • Inhale deeply and hold breath
  • Inhale and exhale quickly
  • Exhale and hold breath

Situation

A nurse is caring for clients with varying gastro-intestinal disorders. One of the clients has biliary atresia. Nausea and vomiting was severe so the doctor ordered for Nasogastric tube placement to decompress the stomach.

A nurse has inserted a nasogastric (NG) tube to the level of the oropharynx and has repositioned the client’s head in a flexed-forward position. The client has been asked to begin swallowing, and as the nurse starts to slowly advance the NG tube with each swallow, the client begins to gag. Which nursing action would least likely result in proper tube insertion and promote client relaxation

  • Pulling the tube back slightly
  • Instructing the client to breathe slowly
  • Continuing to advance the tube to the desired distance
  • Checking the back of the pharynx using a tongue blade and flashlight

Nurse Billy checks the patient’s NGT placement, he knows that the best way to confirm it is

  • Checking for appearance of gastric aspirate
  • Checking the X-ray result
  • Checking for bubbling if immersed in a basin of water
  • Checking for acidic pH of gastric aspirate

The client’s nasogastric (NG) tube stops draining. Which should the nurse implement first to maintain client safety?

  • Verify the tube placement.
  • Instill 30 to 60 mL of fluid.
  • Clamp the tube for 2 hours.
  • Retract the tube by 2 inches.

The patient’s condition improved. Vomiting has lessened but the patient still cannot tolerate oral intake so the doctor made a new order for enteral feeding for nutritional support. A nurse developing a care plan for this client identifies which situations that will place the client at risk for aspiration? Sedation Coughing An artificial airway Head elevated position Nasotracheal suctioning Decreased level of consciousness

The nurse is preparing to initiate bolus enteral feedings via nasogastric (NG) tube to the same client. Which of the following actions represents safe practice by the nurse

  • Checks the volume of the residual after administering the bolus feeding
  • Aspirates gastric contents prior to initiating the feeding and ensures that pH is greater than 9
  • Elevates the head of the bed to 25 degrees and maintains for 30 minutes after instillation of feeding
  • Measures the length of the tube from where it protrudes from the nose to the end and compares it to previously documented measurements

If the patient is on continuous feeding via NGT, how often should the nurse check the tube’s placement

  • Every 6-8 hours
  • Every 1-3 hours
  • Every 4-6 hours
  • Every 8-10 hours

The client’s prognosis went well and the doctor ordered the patient for possible discharge within 24 hours. As the nurse prepares the client for the removal of a nasogastric tube, she instructs the client to

  • Inhale deeply.
  • Exhale slowly.
  • Hold in a deep breath.
  • Pause between breaths.

Situation

Ensuring safety before, during and after a respiratory diagnostic procedure is an important responsibility of the nurse. The following questions apply.

A client named Daryl is suspected of having a pleural effusion. The nurse assesses him for which typical manifestations of this respiratory problem

  • Dyspnea at rest and moist, productive cough
  • Dyspnea on exertion and dry, non-productive cough
  • Dyspnea at rest and dry, non-productive cough
  • Dyspnea on exertion and moist, productive cough

The nurse plans to have which of the following items available for immediate use for Daryl’s untreated condition

  • Intubation tray
  • Paracentesis tray
  • Thoracentesis tray
  • Central venous line insertion tray

The nurse recalls the nursing interventions before, during, and after aspiration of fluid in the pleural cavity. She was not able to recollect accurately if she anticipates that the doctor will not insert the trocar

  • Below the seventh rib laterally and above the ninth rib posteriorly
  • Below the 2nd intercostal space (ICS) anteriorly and above 4th ICS posteriorly
  • Below the seventh rib laterally and below the ninth rib posteriorly
  • Above the 2nd intercostal space (ICS) anteriorly and below 4th ICS posteriorly

The nurse is assessing Daryl’s respiratory status after thoracentesis. The nurse would become most concerned with which of the following assessment findings

  • Equal bilateral chest expansion
  • Respiratory rate of 22 breaths per minute
  • Diminished breath sounds on the affected side
  • Few scattered wheezes, unchanged from baseline

Daryl has become progressively dyspneic and now has been co-diagnosed with left tension pneumothorax. Which of the following observed by the nurse indicates that his pneumothorax is rapidly worsening

  • Tracheal deviation to the left
  • Tracheal deviation to the center of carina
  • Pain on respiration with flat neck veins
  • Tracheal deviation to the right

Pre-Intensive Examination

Situation

Charity, the PHN in the Municipality of Tubog, learned from the residents that children and some elderly have been suffering from respiratory and skin ailments allegedly due to the bad smelling curly dark smoke emitting from the factory nearby. She was invited to the community assembly that was initiated by the barangay council.

The barangay captain asked, “What can you do to help solve the problem of Nurse Charity?” What would be the right response of Nurse Charity?

  • “Well, your problem is easy to solve. I have here some cough syrup, ointments for the skin, and some antibiotics. I will distribute this after the meeting.”
  • “Who among you here have children who are suffering from respiratory and skin diseases? How about the adults who are here? Are you also having the same problems?”
  • “I suppose you gave a lot of thought about the problem and its possible solution. However, treating your children is not the first solution. We have to go to the root cause of the problem.”
  • “May I ask you what solution have you identified for the community problem?”

Situation

Municipality “A” made it to a local newspaper headline because of the occurrence of some unusual events in just 10 days: 2 pregnant women died giving birth at home; 2 children died of dehydration, and 2 elderly died of pneumonia. The mayor of the town called for a meeting of the municipal health office personnel. Public Health Nurse Corazon was assigned to make the action plan to prevent another similar event from occurring.

Which of the following could have been the top contributors to the situation? Select all that apply.

  • Postponement of people in seeking medical care
  • Adherence to cultural practices and beliefs
  • Lack of ambulances and drivers
  • Delays in receiving appropriate and adequate care

Nurse Corazon and her group prepared objectives for plan of action. Which of the following are the best characteristics of a good objective:

  • Futuristic, change-oriented, dynamic and systematic
  • Time-bound, accurate, realistic, measurable, and specific
  • Flexible, accurate, top-priority, and feasible
  • Time-bound, measurable, change-oriented, and continuous

Nurse Corazon presented the plan in a community assembly for acceptance and approval. Which of the following is the most important criterion for the plan’s acceptance and approval by the people?

  • Tasking the community people
  • Volunteerism and willingness of the community people
  • Presence of the municipal health officer in the assembly
  • Huge attendance in the community assembly

To combat the delays in attending to childhood illness, Nurse Corazon adopts the Integrated Management of Childhood Illness in the municipality. This program aims to:

  • Improve and ensure the accessibility and availability of basic and essential health care in both public and private facilities and services.
  • Ensure the quality and affordability of health goods and services.
  • Develop, implement, monitor, and evaluate maternal and newborn care services, ensuring accessibility.
  • The provision of accessible immunization for preventable childhood diseases up to six years of age.

In Municipality A, like any other municipalities, one major problem that makes any health service program inaccessible is:

  • Lack of community awareness
  • Limited number of health-oriented programs
  • Reliance of government financial support
  • Lack of ambulance

Situation

Nurse Kat, a newly promoted senior nurse in the Obstetrics ward, is attending a seminar leadership and management in preparation for her work.

Nurse Kat learns the five principles goal setting in which the senior nurse must provide enough time for OB nurses to improve performance. This principle is:

  • Challenge
  • Commitment
  • Feedback
  • Task Complexity

The nurse also learns that continuous training is a personal and organizational goal. Choose the statements which are true regarding continuous training.

  • Training employees is an excellent investment and a cost to an institution.
  • Continuous training is more of a personal responsibility than institutional.
  • Cross-training and job rotation provide on-going part-time learning experience.
  • Select the best people when hiring employees and invest their retention through continuous training.

Noting the importance of the nurse-patient relationship, Nurse Kat reviewed Hildegard Peplau’s Theory which identified three phases, the first of which is when the pregnant woman:

  • Feels the need to seek professional assistance.
  • Demonstrates self-reliance in caring for herself.
  • Understands the communication of Nurse Kat regarding the services offered.
  • Begins to have the feeling of belonging.

Nurse Kat took note that evaluating the OB staff is an on-going function of management. Some of the reasons for conducting an evaluation includes the following, except for one:

  • To provide an indication of the costs of poor-quality services.
  • To justify the use of resources.
  • To dissuade self-evaluation of the OB staff.
  • To ensure that the quality of care provided by the OB staff.

Situation

You are an OB nurse in an out-patient department. You encounter pregnant women with complications.

A 35-year-old woman, on her second trimester of pregnancy with insulin-dependent diabetes mellitus, comes to you for some advice. What is the priority message for her at this time?

  • Infants of diabetic mothers are big, which can result in a more difficult delivery.
  • Breastfeeding is highly recommended and insulin use is not contraindicated.
  • Achievement of optimal glycemic control is of utmost importance in preventing congenital anomalies.
  • Her insulin requirements will likely increase beginning the third trimester of pregnancy.

A 30-year-old G6P5 woman at 12 weeks has just begun prenatal care. Her initial laboratory reveals that she has a human immunodeficiency virus (HIV) infection. What would be the priority evidence-based nurse information for this patient?

  • Breastfeeding is still recommended due to the great benefits to the infant.
  • Pregnancy is known to accelerate the course of HIV disease in the mother.
  • Medication for HIV infection is safe and can greatly reduce transmission of HIV to the infant.
  • Breastfeeding will potentiate the transmission of HIV from the mother to the child.

Situation

The family is the unit of service in community health nursing. Public Health Nurse Olive takes care of the Emmanuel Family. Jay, the father, works as a janitor in the municipal hall. His wife, Lynn, tends to a small variety store in front of their house. Their children are Lynnel (6 y.o.), Emalyn (5 y.o.), Lynnette (4 y.o.), and Manilyn (3 y.o.).

Based on the characteristics of the family, Nurse Olive can easily categorize the family’s as being in which development stage?

  • Families with pre-school children
  • Family with school-age children
  • Beginning family
  • Early child-bearing

In this particular stage, which of the following is a most basic concern?

  • Emergence of harmony in marital and in-law relations
  • Child-rearing
  • The couple shifts to adult social interests
  • Balancing time and energy to meet the demands of work and family

As a young manager, Freda knows that conflict occurs in any organization. Which of the following statements regarding conflict is not true?

  • It may result to poor performance of the staff.
  • It can be destructive, hence, it should not reach the highest level.
  • It is not beneficial, hence, it should be prevented at all times.
  • It may create a new leader from among the staff.

Freda tells one of the staff: “I don’t have time to discuss the matter with you at this moment,” when the latter asks if they can talk about an issue. Which of the conflict resolution strategies did she use?

  • Avoidance
  • Smoothing
  • Compromise
  • Restriction

Which of the following is the best action that Freda can take?

  • Quit her job and look for another employer.
  • Disregard what she feels and continue to work independently.
  • Identify the source of conflict and understand the source of friction.
  • Seek help from the director of nursing.

Freda knows that one of her staff is experiencing burnout. Which of the following is the most appropriate course of action?

  • Remind her to show loyalty to the institutions.
  • Ignore her observations; it will be resolved even without intervention.
  • Let her staff vent their feelings and ask how she can be of help.
  • Advise her staff to go on vacation.

Freda knows that the performance appraisal consists of the following activities, except:

  • Setting specific standards and activities for individual performance.
  • Using agency standards as a guide.
  • Focusing activity on the correction of identified behavior.
  • Determine areas of strengths and weaknesses.

Situation

Nurse Dora, a nursing staff applicant, passed both written and oral examinations. Because she knows the head of office, she promised to submit all her credentials after she has “fixed things up”. She was appointed as Nurse I with a temporary status until she submits all her credentials, including her PRC license. Her evaluation performance was satisfactory. After a year, however, she had to renew her PRC registration and identification (ID) card.

What action must the nursing administration do initially?

  • Report the matter to the head of office who has the description to appoint the nurse.
  • Verify with the professional regulation commission regarding the status of Nurse Dora.
  • Confront Nurse Dora and terminate her.
  • Write a letter to the Civil Service Commission for proper action for Nurse Dora.

It was found out that Nurse Dora did not pass the Nurse Licensure Examination (NLE). What legal action should be filed against her?

  • Dishonesty
  • Conduct unbecoming of a professional
  • Malpractice
  • Misrepresentation

In case Dora committed a medication error during her duty, the head of office may be held liable because of the principle of:

  • Unethical conduct
  • Respondeat superior
  • Politicking
  • Res Ipsa Loquitur

All nurses must understand that, after graduation, they must pass the NLE. To be registered in the roster, they should take the Professional Oath with the following, except:

  • Member of Sangguniang Panlalawigan
  • Governor of Philippines Nurse Association
  • Member of the Professional Regulatory Board of Nursing
  • Provincial Governor

A nurse is leading a community meeting at a partial-hospitalization program. One group member talks constantly and interrupts the other patients. The most appropriate action for the nurse to take is to:

  • Explain to the member, after the meeting, that the group time should be shared.
  • Thank the group members for his contribution and ask the other members for their ideas.
  • Ask the group members if they are satisfied with the way the group is working.
  • Remain silent and wait for another group member to speak up.

Situation

Arlene, 21 years old, is a law graduate. She wants to review for the bar examination, but she is pregnant. She said she has been experiencing regular menstruation but does not know when ovulation usually occurs. This has something to do with her fertility period during her last sexual intercourse with her husband.

As a nurse, what would you tell Arlene regarding ovulation? The ovulation usually corresponds to the life of the corpus luteum which occurs approximately:

  • 14 days after the first day of the succeeding menstrual period.
  • 7 days after the first day of the succeeding menstrual period.
  • 7 days before the first day of the succeeding menstrual period.
  • 14 days before the first day of the succeeding menstrual period.

Arlene insisted that she must have been fertile during the time of sexual intercourse. The nurse explains that the absolute period of fertility is the span of time that a woman is likely to be pregnant when she engages in unprotected sexduring which period?

  • Several days after ovulation
  • During ovulation
  • Immediately after ovulation
  • Immediately before ovulation

The nurse proceeded to take the menstrual history of Arlene to find out if she is likely to be pregnant. Which of the following determines the date of onset of the last menstrual period (LMP)?

  • Duration and character of the LMP
  • Implantation bleeding
  • Spotting after the LMP
  • Bleeding before the LMP

The nurse also asked about Arlene’s secondary amenorrhea that would most likely indicate pregnancy. Secondary amenorrhea is the cessation of menstrual bleeding for more than how many months after the regular menstrual cycle has been established?

  • Five
  • Three
  • Six
  • Four

The nurse also asked for the presence of secondary dysmenorrhea. Which of the following conditions is included as a cause of under secondary dysmenorrhea?

  • Intrauterine Device (IUD)
  • Pelvic Inflammatory Disease (PID)
  • Malposition of the cervix
  • Absence of any underlying anatomic abnormality

Situation

Jaina is a fourth-year nursing student currently in community immersion. She, together with her instructor, is conducting a home visit with one of the patients, who is diagnosed with open-angle glaucoma.

In the middle of their conversation, the client raised a question to Jaina. She mentioned that she has been prescribed with Latanoprost (Xalatan) by the municipal doctor. She asks Jaina why she has to take this.

  • The medicine dilates the affected eye
  • The medicine moistens the affected eye
  • The medicine decreases the angle of glaucoma in her affected eye
  • The medicine decreases the intraocular pressure in her affected eye

In utilizing the IMCI protocol, the nurse should initially:

  • Observe the condition of the child
  • Ask the mother what the problem of the child is
  • Look for danger signs
  • Identify the main symptoms

Situation

The local health board established a reproductive health clinic in the main health center. Two nurses, Hubert and Irene, were assigned to handle services to address problems related to sexuality, reproductive health, and fertility problems.

Nurse Valentin classifies cases according to the major categories of reproductive tract infections. Which of the following is not part of such classifications?

  • Iatrogenic infections as an aftermath of invasive procedures like catheterization and intrauterine device (IUD) insertion
  • Urinary tract infections among male and female patients
  • Sexually-transmitted infections
  • Endogenous infections resulting from poor personal hygiene

Irene handles the screening for gonorrhea every two weeks among female sex workers in the implementation of PD 856. In differential diagnosis of discharge among infected clients, which of the following colors of discharge will Irene take note to identify gonorrhea from other causes?

  • Greenish-yellow as differentiated from mucoid white of trichomoniasis
  • Mucoid white as compared to graying-white discharge of vaginosis
  • Graying-white as differentiated from mucoid-white of chlamydia
  • Yellowish-white as compared to greenish-yellow discharge of trichomoniasis

Nurse Valentin was invited by a women’s group to give a lecture on healthy sexuality. In the expectation check, he noted that there are previous misconceptions expressed by the participants. Which of the following statements are correct?

  • It is the obligation of the wife to give in to sex whenever the husband requests it.
  • Sexuality is fluid and may change.
  • Effeminate men are gays.
  • Homosexuality is an abnormality.

One of the clients has gonorrhea. Nurse Irene explained that gonorrhea and chlamydia, if left untreated, can lead to pelvic inflammatory disease (PID). Such a condition may result in infertility due to:

  • Foul-smelling discharge resulting in the death of ovum
  • An unknown cause
  • Scarring which can lead to tubal occlusion
  • Purulent discharge which can kill the sperm

The nurse further explained that a test used to determine tubal patency using a radiopaque material is called:

  • Post-coital infertility test
  • Sims-Huhner Test
  • Friedman’s Test
  • Hysterosalpingography

Situation

Sandy is a newly hired nurse as the 3 to 11 duty nurse at the Neonatal Intensive Care Unit (NICU). She received an endorsement that Baby Boy A and Baby Girl B are ready for discharge. Hours later, Mother A returns to complain because she finds that the baby she received was not hers.

Which of these may prove Nurse Sandy may be charged for gross negligence? Select all that apply.

  • That Nurse Sandy held the responsibility.
  • That Nurse Sandy failed to fulfill that responsibility.
  • That, due to her actions, a patient was injured.
  • That there is a causal relationship between the failure of the nurse and the resulting injury.

What could have been done to prevent the occurrence of baby-switching?

  • Calling out the name of the mother
  • The use of proper name tags for identification
  • Checking the genitalia
  • Identifying the babies by checking their footprints

The most important element that determines Nurse Sandy was incompetent is that:

  • She is a newly-hired nurse.
  • She did not do her job according to her orientation.
  • She lacked supervision.
  • She did not do her job according to a set standard.

Which of the following should Sandy understand as the primary outcome of being a professional nurse should she be proven guilty of incompetence?

  • That she be delisted as a member of the accredited professional organization
  • That she can be terminated from work by her employer
  • That she can be charged in the civil court
  • That her license to practice nursing can be suspended or revoked for a period of time by the Professional Regulation Commission

Which among the statements is true about health promotion?

  • It is the process of enabling individuals and communities to increase their abilities to control and improve their health.
  • It is the process of enhancing the individual’s physical, psychologic, and spiritual well-being.
  • It is the process of engaging in correcting the people’s health behavior.
  • It is the process of preventing disease from occurring and treating it when it occurs.

A number of factors influence the health status of the people in the community. A category of these include health care system influences. Which of the following do not fall under this category?

  • Community resources and programs that are available
  • The cost of health care
  • Community perception of the value of available health care
  • The availability of technology

Which of the following health behavior choices are essential to promoting health and preventing disease?

  • Taking prescribed medications at the right time and dosage.
  • Eating the right kind of food, adequate sleep, physical exercise, and effective handling of stress.
  • Refraining from travelling to countries with high prevalence of infectious disease.
  • Smoking cessation and drinking alcohol regularly.

There are behaviors that are encouraged by cultural expectations. Which is not included?

  • Taking herbal supplements to boost the immune system.
  • Relying for information from community resources/programs.
  • Having a therapeutic massage (“hilot”) at least once a month during pregnancy.
  • Talking openly about the details of illness.

What are the essential factors in the integrative models of human health?

  • Intertwined and interactive, with multiple components such as physical, psychological, and social
  • Effective only when combined and integrated with alternative therapies
  • Generally being attributed to psychological problems with individuals
  • Derived solely from physical phenomenon

Situation

Nurse Cassie recently passed the NLE. She is currently practicing as a CHN nurse in Barangay Alde wherein keen assessment and practical nursing skills are much needed.

Not having a regular check-up is an example of:

  • Health Problem
  • Health Need
  • Health Threat
  • Health Deficit

A condition that promotes disease or injury and prevents people from realizing their health potential is a:

  • Health Problem
  • Health Need
  • Health Threat
  • Health Deficit

A situation in which there is a demonstrated health need combined with actual or potential resources to apply remedial measures and a commitment to act on part on the part of the provider or the client is a:

  • Health Problem
  • Health Need
  • Health Threat
  • Health Deficit

A lack of medical or social technology is an example of:

  • Health Problem
  • Health Need
  • Health Threat
  • Health Deficit

Stressful occurrences are examples of:

  • Stress Point
  • Health Issue
  • Foreseeable Crisis
  • Critical Point

In the public health bag, how many pair/s of forceps are needed?

  • None
  • 1
  • 2
  • 3

In the bag technique, how should the nurse handle the apron?

  • Take out the apron from the bag and put on the up side out
  • Take out the apron from the bag and put on with the down side out
  • Take out the apron from the bag and put on with the left side out
  • Take out the apron from the bag and put on with the right side out

The following items are not part of the public health bag, except for:

  • Hypodermic needles
  • Candle
  • Flashlight
  • Ruler

Which of the following solutions are not found in the public health bag?

  • Zephiran solution
  • 40% alcohol
  • Benedict’s solution
  • Spirit of ammonia

The following pairs of scissors are not found in the public health bag, except for:

  • Surgical Scissors
  • Bandage Scissors
  • Beauty and Care Scissors
  • None of the above

Situation

The family of Roxas is fond of dogs. A vendor who entered their resident without notice is bitten by one of their pets. PHN Carl attends to the vendor.

Which of body part of the vendor will be most affected by rabies?

  • Buttocks
  • Head
  • Feet
  • Hand

To protect the vendor from the danger of rabies, PHN Carl advises him to clean the wound thoroughly with soap and water, consult a physician, and receive anti-rabies vaccination. Which among the following vaccines can provide active immunity?

  • Purified vero cell vaccine
  • Human rabies immunoglobulin
  • Equine rabies immunoglobulin
  • Purified duck embryo vaccine

The vendor acquired rabies. What will PHN Carl do to protect those who took care of him? He should administer:

  • Pre-exposure prophylactic treatment for the family of the vendor
  • Pre-exposure prophylactic treatment to the dog and the vendor’s family members
  • Post-exposure prophylactic treatment for the family of the vendor
  • Post-exposure prophylactic treatment to the dog and the vendor’s family members

PHN Carl’s intervention to protect all residents who own pets, especially dogs, should be done by:

  • Coordinating with the city or municipal agriculturist for the immunization of all pets
  • Coordinating with the city or municipal’s officials to make an ordinance on stray dogs
  • Massive campaign to families not to own pets at home
  • Massive campaign for responsible pet ownership

Situation

The Field Health Services and Information System (FHSIS) is a recording and reporting system for public health care in the Philippines.

The following are the objectives of the FHSIS, except for:

  • Complete the picture of acute and chronic disease
  • Ensure data recorded are useful and accurate, and are disseminated in a timely, easy-to-use fashion
  • Minimize recording and reporting burden allowing more time for patient care and promotive activities
  • Providers standardized facility-level database which can be used for more in-depth studies

As a nurse, you should know the process of how the information is processed and consolidated. The fundamental unit of the FHSIS system is the:

  • Family Treatment Record
  • Output Record
  • Reporting Forms
  • Target/Client List

The monthly field health service activity report is a form used in which of the components of the FHSIS?

  • Target/Client List
  • Output Report
  • Individual Health Record
  • Tally Report

In using the tally sheet, the recommended frequency in tallying activities and services is?

  • Weekly
  • Quarterly
  • Monthly
  • Daily

To monitor clients registered in long-term regimen such as multidrug therapy, which component of the reporting system will be most useful?

  • Output Report
  • Tally Report
  • Target/Client List
  • Individual Health Record

Situation

Mel, Kid, Emma, Gliz, and Trix are close friends in the same age range (8 to 9 y.o.), as orphaned, and out-of-school street youths. Nurse Grace from the Department of Social Welfare and Development (DSWD) is tasked to attend to them.

The nurse’s responsibility is to check on the children’s status and patterns. Which of the following would the nurse’s priority?

  • Interaction Patterns
  • Health Status
  • Adaptation Status
  • Developmental Patterns

What could be the most important health threat for these children?

  • Infection
  • Depression
  • Malnutrition
  • Injury

What should be the primary goal of Nurse Grace for these street children?

  • To change the behavior of the children
  • To analyze the information gathered
  • To locate the family of the children
  • Elicit important data about the family

What is the best approach to discuss the prevention of health threats to the children?

  • Focus Group Discussion
  • According to gender
  • One-on-one interaction
  • According to age

As the facilitator/leader of the discussion on staying away from conflicts with each other and other street children, which of the following tasks should be induced by Nurse Grace?

  • Listening attentively to what each child is saying
  • Doing away with blaming and ridiculing
  • Forcing all the street children to participate in the discussion
  • Being sensitive in identifying children who are eager to talk

Situation

As a CHN nurse, Nurse Jake is responsible for home visits in Town A. Thus, he needs to have a strong foundation in blood pressure monitoring and knowledge regarding communicable diseases such as HIV.

In the preparatory phase of blood pressure management, how many minutes should the client have been relaxed and how many minutes should the client have not smoked or ingested caffeine?

  • 10 minutes; 15 minutes
  • 5 minutes; 30 minutes
  • 5 minutes; 15 minutes
  • 10 minutes; 30 minutes

In applying the blood pressure cuff and stethoscope, how many centimeters above the brachial artery should the nurse apply the cuff?

  • 2 to 3 centimeters
  • 3 to 4 centimeters
  • 4 to 5 centimeters
  • 5 to 6 centimeters

For obese persons, which part of the stethoscope is preferably used to auscultate the pulse in obtaining the blood pressure reading?

  • The part of the stethoscope used is irrelevant.
  • The bell
  • The diaphragm
  • Both parts are preferable in obese persons.

Nurse Jake knows that he should take the mean of two readings in order to have an accurate reading. How many minutes will he wait in between the two readings?

  • 5 minutes
  • 4 minutes
  • 3 minutes
  • 2 minutes

Which situation would require Nurse Jake to obtain a third reading?

  • If the first two readings differ by 15 mm Hg.
  • If the first two readings differ by 10 mm Hg.
  • If the first two readings differ by 5 mm Hg.
  • If the first two readings do not have any difference.

In handling the soiled linens and clothes of an HIV patient, how should Nurse Jake proceed?

  • Fold the wet portions towards the dry portions
  • Fold the dry portions towards the wet portions
  • Fold corners towards the middle
  • Fold in a circular manner towards the middle

A family member of a diseased HIV client asks Nurse Jake if it is possible to have the body available for public viewing.

  • “No, the body must be disposed immediately.”
  • “Yes, the embalmed cadaver is safe for public viewing.”
  • “Please ask the physician regarding this matter.”
  • “No, only family members should be allowed to view the body.”

Which of the following procedures done to an HIV-positive client is an example of a low-risk procedure?

  • Oral surgery
  • Venipuncture
  • Collection and transport of laboratory specimen
  • Injections

Oral prophylaxis is an example of a:

  • Minimum-risk procedure
  • Low-risk procedure
  • Moderate-risk procedure
  • High-risk procedure

The following procedures are not examples of moderate-risk procedures for a client with HIV except for:

  • Lumbar puncture
  • Grafting
  • Phlebotomy
  • All of the above

Situation

Nurse Marco has recently become a member of the Occupational Health Nurses Association of the Philippines (OHNAP). He understands the risk factors and determinants of health when concerned with occupational hazards.

As an occupational health nurse, Nurse Marco knows that the following workers are at risk:

  • Older, men, self-employed workers
  • Older, women, self-employed workers
  • Older, women, employed in larger organizations
  • Older, women, employed in smaller organizations

Injury rates tend to be higher in which kinds of organization?

  • There is no relation between injury rates and organizational size.
  • Smaller (<50)
  • Mid-size (50 to 249)
  • Larger (>249)

The following are leading causes of fatal work injuries except:

  • Falls
  • Homicides
  • Highway accidents
  • None of the above

Any injury that results from a single incident in the work environment is a:

  • Occupational Illness
  • Occupational Injury
  • Options 1 and 2 are interchangeable.
  • Occupational Crisis

Any abnormal condition or disorder caused by exposure to environmental factors associated with employment is a:

  • Occupational Illness
  • Occupational Injury
  • Options 1 and 2 are interchangeable.
  • Occupational Crisis

Situation

The Milk Code of the Philippines (EO 51) regulates the marketing of infant milk formula, other milk products and beverages, as well as feeding bottles and teats. It is one of the strongest breastfeeding protection laws in the world, and makes way for the promotion of the benefits of breastmilk over formula.

Which statement is not a benefit of breastfeeding for the infant?

  • Increases IQ points
  • Reduces infant exposure to infections
  • Safely dehydrates and provides nutrients to a sick child, especially to those suffering from diarrheal diseases
  • Strengthens the infant’s immune system, preventing many infections

Which benefit of breastfeeding is not directly for the mother?

  • Reduces risk of excessive blood loss after giving birth
  • Provides nutritionally-complete food
  • Provides natural methods of delaying pregnancies
  • Reduces the risk of ovarian and breast cancers, and osteoporosis

The household and the community may benefit from breastfeeding by:

  • Conservation of funds that otherwise would be spent on breastmilk substitutes, supplies, and fuel to prepare them.
  • Saving of medical cost to families sand governments by preventing illnesses and by providing immediate postpartum* and contraception.
  • None of the above
  • Options 1 and 2

Situation

As a nurse, adequate knowledge of research is integral for the development of nursing science.

This refers to the integration of research findings where the practical application is related to the original research. It utilizes the PICO technique in answering the clinical question.

  • Research utilization
  • Research dissemination
  • Evidence-based practice
  • Cochrane collaboration

You are a nurse and you wanted to study the relationship between quality of life of seafarers and the use of social media while on-board. Which is the independent variable?

  • Use of social media
  • Quality of life of seafarers
  • On-board
  • Seafarers

Based on the previous question, which is the outcome variable?

  • Use of social media
  • Quality of life of seafarers
  • On-board
  • Seafarers

A nurse is preparing a research proposal wherein she will compute the sample size from a population of 2,340 with a margin of error of 5%. What is the computed sample size using Slovin’s formula

  • 1,134
  • 1,563
  • 342
  • 565

A nurse research conducts case control study on the association of physical activity and development of acute coronary syndrome (ACS) and found out an odds ratio of 0.83. The nurse correctly infers that:

  • People with ACS were 0.83 times more likely to be physically active than those who do not have ACS.
  • Risk of ACS among those who are physically active is 83% compared to those who are not physically active.
  • 83 cases of ACS are attributable to physical activity.
  • 83% of patients who have ACS have a history of adequate physical activity.

A public health nurse recorded 15 new cases of dengue fever for the months January to June 2018. Currently, 20 are being treated at the health center since last year. What is the incidence considering a population at risk of 95,000?

  • 22 per 100,000 for the year 2017
  • 11 per 100,000 for January to June 2018
  • 16 per 100,000 for January to June 2018
  • 9 per 100,000 for July to December 2017

In a research proposal, a nurse mentioned that snowball sampling will be used as a sampling method. The nurse understands that they will select samples by:

  • Asking sample members to refer other people who meet the eligibility criteria.
  • Dividing the population into subsets from which elements are selected at random.
  • Using the researcher’s knowledge about the population to hand-pick sample members.
  • Establishing a sampling frame where the sample members are chosen at random.

A nurse is studying the defense mechanisms of young adults towards the problems they encounter during their first jobs. Which type of research design is best to use?

  • Quantitative
  • Retrospective
  • Qualitative
  • Experimental

A nurse is studying the defense mechanisms of young adults towards the problems they encounter during their first jobs. Which type of research design is best to use?

  • Quantitative
  • Retrospective
  • Qualitative
  • Experimental

In their research, nurses found out that children living in Manila urban poor areas are vulnerable to contracting pulmonary tuberculosis due to housing congestion. What kind of research design is utilized?

  • Qualitative
  • Quantitative
  • Survey
  • Mixed Methods

A head nurse of a medical ward is planning to convince her staff nurses to start conducting nursing research to improve nursing practice. Which of the following is the easiest way to do this?

  • Ask them to join in data gathering
  • Encouraging them to read related literature regarding the study.
  • Let them analyze the data.
  • Instruct them to run statistical tests on the data gathered.

Using the Pearson Product Moment Correlation, the statistical test yielded a -0.75 Pearson’s value. How does the nurse interpret this?

  • High positive correlation
  • High negative correlation
  • Moderate positive correlation
  • No significant association

Recalls I

This exam was (mostly) rationalized by Ms. Julianne Lupisan, RN.

Situation

Public Health Nursing in the Philippines evolved alongside the institutional development of the Department of Health, the government agency mandated to protect and promote people’s health and the biggest employer of health workers including public health nurses.

In response to above trends, the global community, represented by the Nations General Assembly, decided to adopt a common vision of poverty reduction and sustainable development in September 2000. This vision is exemplified by the Millennium Development Goals (MDGs) which are based on the fundamental values of freedom, equality, solidarity, tolerance, health, respect for nature, and shared responsibility. Which among these is not a Millennium Development Goal?

  • Improve maternal health
  • Combat HIV/AIDS, malaria and other diseases
  • Ensure environmental sustainability
  • Promote gender equity and empower women

Factors or things that make people healthy or not, known as determinants of health are listed by the World Health Organization to include all except one:

  • Genetics
  • Gender
  • Location
  • Culture

According to C. E. Winslow, “Public health is the science and art of (1) preventing disease, (2) prolonging life, and (3) promoting health and efficiency through organized community effort.” Which is considered as the key phrase definition of public health?

  • development of the social machinery to ensure everyone a standard of living adequate for the maintenance of health
  • through organized community effort
  • organization of medical and nursing services for the early diagnosis and preventive treatment of disease
  • education of the individual in personal hygiene

Public health nursing is strongly built on theories of health promotion and disease prevention. This model found that information alone is rarely enough to motivate people to act for their health. Individuals must know what to do and how to do it before they can take action.

  • Health Belief Model
  • Health Promotion Model
  • Social Learning theory
  • General Systems Theory

It is based on the assumption that behavior change takes place over time, and progresses through stages. Each stage is stable and is open to change; meaning one may stop in one stage, progress to the next stage or return to a previous stage.

  • General Systems Theory
  • Transtheoretical Model
  • PRECEDE-PROCEED Model
  • Milo’s Framework for Prevention

Situation

WHO defines PHC as essential health care made universally accessible to individuals and families in the community by means acceptable to them through their full participation and at a cost that the community and country can afford at every stage of development.

What is considered the legal basis of PHC in the Philippines?

  • LOI 949
  • LOI 497
  • LOI 789
  • LOI 747

This is signed by which President of the Philippines?

  • Pres. Ferdinand Marcos
  • Pres. Gloria Aquino
  • Pres. Ramon Magsaysay
  • Pres. Ferdinand Marcos Jr.

The PHC has 8 essential health services in which it aims to improve. All of these are included, except:

  • Maternal and Child health
  • Nutrition
  • Exclusive Program for Immunization
  • Education for health

Considerations are made regarding the health services being provided to the people. These have to be to be available, acceptable, affordable and accessible to the people to reach the success of Primary Health Care. Which criteria questions whether the health service is offered in health care facilities or is provided on a regular and organized manner?

  • Acceptability
  • Affordability
  • Accessibility
  • Availability

The PHC uses multisectoral approaches to communicate, collaborate, and convene to achieve universal health care. When the doctor has to refer the victim of Violence Against Women to the Department of Social Welfare and Development, what referrals or linkages will they use?

  • Intrasectoral linkages
  • Intersectoral linkages
  • Interfacility linkages
  • Intrafacility linkages

Situation

Kong is learning to use the available resources in Brgy. Kagalingan in providing care to his patients. He firmly believes in the efficiency in using traditional and alternative medicine in his plan of care so that it will be easier for the people to accept and follow.

A 7-year old child is being accompanied by his mother to the clinic. The child has been experiencing abdominal pain, and is presenting with a round belly. His mother said that he did drink any “pampurga” when he was young. What medicinal plant should he give?

  • Lagundi
  • Yerba Buena
  • Tsaang Gubat
  • Niyog-niyogan

How would King Kong tell the mother to prepare and administer the medicinal plant?

  • Decoction
  • Leaves are eaten
  • Poultice
  • Seeds are used

People at Brgy. Kagalingan often complains having pain in their joints after regularly eating internal organs and beans. What can he advise to the people to take?

  • Sambong
  • Bayabas
  • Ulasimang bato
  • Akapulco

This medicinal plant is used to help lower hypertension and blood cholesterol.

  • Ampalaya
  • Bawang
  • Akapulko
  • Sambong

Type II diabetes is also rampant in the barangay. He conducts group health teachings to include which plant into their diet?

  • Ampalaya
  • Bawang
  • Akapulco
  • Sambong

Situation

The Centers for Disease Control and Prevention (CDC) established an office in the Philippines in 2022 to strengthen and expand existing partnerships with the Philippines Department of Health (DOH).

This vital statistics indicate the natural decrease in population.

  • Crude birth rate
  • Crude death rate
  • Prevalence Rate
  • Incidence rate

Nurse Caloy wants to determine the health status of a population and infer their longevity of life and lifestyle by using statistics. What vital statistics shall he use?

  • Prevalence Rate
  • Swaroop’s Index
  • Maternal Pregnancy Rate
  • Case Fatality Ratio

This statistics indicates the health status of the community.

  • Fetal death rate
  • Neonatal mortality rate
  • Infant mortality rate
  • Mortality rate

This is also known as the index of pregnancy wastage.

  • Fetal death rate
  • Neonatal mortality rate
  • Infant mortality rate
  • Mortality rate

Retrieving this number, this will help direct the efforts to improve maternal post partum care in the community. This also shows that an improvement in environmental factors must be promoted to improve the numbers.

  • Fetal death rate
  • Neonatal mortality rate
  • Infant mortality rate
  • Mortality rate

Situation

In Community Health Nursing, the community is not simply a context of the existence of the nurse’s clients nor is it just a setting for our nursing interventions. Spradley (1990) emphasizes that the community is the primary client for two main reasons. First, the community has a direct influence on the health of the individual, families, and sub-populations. Second, it is the level that most health service provision occurs.

According to Freeman and Heinrich, community health diagnosis is based on three interdependent, interacting and constantly changing conditions. Which of these is not included?

  • The health status of the community, including the population’s level of vulnerability
  • Diversity within the community, focusing on making comprehensive and inclusive community health care
  • Community health capability or the ability of the community to deal with its health problems
  • Community action potential, or the patterns in which the community is likely to work on its health problems

Collecting data is important in community assessment. There are different methods as to how one can obtain data. Which method is used to elicit and explore opinions of people, determine their attitudes and practices regarding limited set of concepts? Participants selected should have characteristics that will be common to them, and characteristics that will also differentiate them from one another.

  • Group interviews
  • Key Informant Interviews
  • Focus Group Discussions
  • Windshield Survey

You wanted to be as comprehensive as possible in identifying different problems in the community. You have identified that the RHU is having difficulty in providing services to their population coverage. Further assessment shows that there are a lot of plantilla positions left unfilled. What kind of problem is this?

  • Health status problem
  • Health resources problem
  • Health-related problem
  • Health concern problem

You have also observed that the local government does not make health as their priority thus leaving their BHCs and RHUs lacking of resources and manpower. There is also a lack health program implementation as no support is received. This is classified as:

  • Health status problem
  • Health resources problem
  • Health-related problem
  • Health concern problem

Reviewing the records, you found out that there is an increasing number of non-communicable diseases in the barangay. You classify this problem as?

  • Health status problem
  • Health resources problem
  • Health-related problem
  • Health concern problem

Situation

Nurse Jessa attempts to identify the possible factors associated with the increasing cases of malnutrition in Brgy. Hintuturo.

She attempts to derive the risk of an individual developing a disease due to a particular exposure by comparing the occurrence of a disease in a population exposed to the suspected factor. She wants to use an estimate which is the direct measure of strength of association between a suspected cause and effect. Her existing data includes the incidence rates of those who had who were exposed and incidence rate among those who are not exposed. Which risk estimate shall she use?

  • Odds Ratio
  • Attributable Risk
  • Incidental Risk Ratio
  • Relative Risk

Using the PRECEDE-PROCEED model, she is also aiming to identify the contributing factors of a problem that may become the focus of subsequent intervention. These are characteristics of the client that motivates behavior related to health, and it can be described in terms of client’s knowledge, attitudes, values and perceptions.

  • Predisposing factors
  • Internal factors
  • Enabling factors
  • Reinforcing factors

These are considered the factors which can be rewards or punishments following or anticipated as a consequence of a health behavior.

  • Predisposing factors
  • Internal factors
  • Enabling factors
  • Reinforcing factors

As the program comes to an end, she plans to look into the three aspects of the program. She wants to measure the immediate effects of the program and determine whether the objectives of the program were met. What evaluation measure will she use?

  • Process evaluation
  • Impact evaluation
  • Structure evaluation
  • Outcome evaluation

In evaluating programs, we use indicators to identify which factors should be achieved and resolved by the community. Using the data information gathered from the indicators can be used as basis for recommendations on the future and direction of health programs to be evaluated. Which indicator shows to what extent something that has been made available is actually being used for that purpose?

  • Relevance
  • Coverage
  • Efficiency
  • Utilization

Situation

Nurse Dan is handling patients with respiratory problems. Tommy has been diagnosed with ineffective airway clearance due to excessive secretions and is at risk of infection from these retained secretions. As part of Nurse Dan’s care plan, the goal is to loosen and remove these excessive secretions from Tommy’s airway.

After listening to Tommy’s lung sounds, Nurse Dan identifies congestion in the upper lobes. To drain the anterior and posterior apical segments of the lungs during percussion, the correct position would be:

  • Client lying on his back, then flat on his abdomen in the Trendelenburg position
  • Client seated upright in bed or on a chair, then leaning forward while sitting, then flat on his back and abdomen
  • Client lying flat on his back, then flat on his abdomen
  • Client lying on his right side, then left side in the Trendelenburg position

When documenting the results of Tommy’s treatment, Nurse Dan should include the following details EXCEPT:

  • Color, amount, and consistency of sputum
  • Character of breath sounds and respiratory rate before and after the procedure
  • Amount of fluid intake before and after the procedure
  • Significant changes in vital signs

When evaluating Tommy for chest percussion, vibration, and postural drainage, Nurse Dan should concentrate on the following EXCEPT:

  • Amount of food and fluid consumed during the last meal before treatment
  • Respiratory rate, breath sounds, and location of congestion
  • Instructing the client’s relatives on how to perform the procedure
  • Doctor’s orders regarding position restrictions and the client’s ability to tolerate lying flat

When preparing Tommy for postural drainage and percussion, which of the following is a special consideration?

  • Respiratory rate of 16 to 20 per minute
  • Client’s ability to tolerate sitting and lying positions
  • Absence of signs of infection
  • Time of the client’s last food and fluid intake

The purpose of chest percussion and vibration is to loosen lung secretions. The difference between these procedures is:

  • Percussion uses only one hand, while vibration uses both hands
  • Percussion delivers cushioned blows to the chest with cupped palms, while vibration gently shakes secretions loose during the exhalation cycle
  • In both percussion and vibration, the hands are placed on top of each other and move in sync with the client’s breathing rhythm
  • Percussion slaps the chest to loosen secretions, while vibration shakes the secretions during inhalation

Situation

Nurse Angel is doing her rounds in the ward, when someone called a code. She rushed immediately to the bedside.

Nurse Angel grabs the defibrillator and rush to the room where resuscitation is in progress. A colleague has started CPR and confirms the patient is in ventricular tachycardia. As she begins to attach the AED, she notices a transdermal medication patch on the victim’s upper right chest, exactly where she needs to place an AED electrode pad. What is the most appropriate action?

  • Ignore medication patch and place the electrode pad in the standard position
  • Avoid medication patch and place second electrode pad on victim’s back
  • Remove the medication patch, wipe the area dry, and place the electrode pad in correct position
  • Place electrode pad on the victim’s right abdomen

It is important for her to recognize which rhythms are for cardioversion, defibrillation or non-shockable. Which among the following is non-shockable?

  • Ventricular fibrillation
  • Ventricular tachycardia
  • Asystole
  • Pulseless ventricular tachycardia

This is considered as the most important intervention for VF/ pulseless VT with the greatest effect on survival to hospital discharge?

  • Epinephrine
  • Defibrillation
  • Oxygen
  • Amiodarone

Which of the following is the most important step to restore oxygenation and ventilation for an unresponsive, breathless near-drowning victim?

  • Attempt to drain water from the breathing passages by performing the Heimlich maneuver
  • Begin chest compressions
  • Provide cervical spine stabilization because a diving accident may have occurred
  • Open the airway and begin rescue breathing as soon as possible, even in the water

In initiating CPR, nurse Angel knows it is important to do chest compressions correctly. What is the correct way of performing chest compressions?

  • Compress more than 120 per minute
  • The rescuer’s arms should be parallel to the victim’s body
  • Compress for 5 cycles at 30 compressions each cycle
  • Depth of compressions should be 4 inches

Situation

Proper positioning is important to conduct nursing activities.

The best position for any procedure that involves vaginal and cervical examination is

  • Dorsal recumbent
  • Side lying
  • Supine
  • Lithotomy

Which of the following is the best position for the client to assume if the back is to be examined by the nurse?

  • Standing
  • Supine
  • Side lying
  • Prone

In palpating the client’s abdomen, Which of the following is the best position for the client to assume?

  • Dorsal recumbent
  • Side lying
  • Supine
  • Lithotomy

Rectal examination is done with a client in what position?

  • Dorsal recumbent
  • Sims position
  • Supine
  • Lithotomy

One of the patients diagnosed with COPD is having difficulty breathing. Which is the best position to achieve maximal lung expansion?

  • High fowlers
  • Semi fowlers
  • Supine
  • Orthopneic

Situation

Understanding the factors that influence the healthcare delivery system enables nurses to adapt to changes, devise improved methods of providing care, and develop new nursing roles.

Cost-effective healthcare prioritizes the primary prevention of illness. Which of the following is an example of a primary prevention activity?

  • Administering first aid treatment for a patient bitten by a snake
  • Providing home modifications to help a client adapt their home environment following a stroke
  • Offering a weekly ZUMBA class to young adults with a strong family history of high cholesterol
  • Vaccinating children with anti-rabies after being bitten by a stray dog

The Cardinal Santos Medical Center often holds free health programs in the outpatient department. This month of May, they are holding free Pap smears to the first 10 registrants each day. This is an example of what kind of prevention?

  • Primary
  • Secondary
  • Tertiary
  • Quaternary

Prior to discharge, nurse Joey discusses with the Santos family the diet modifications their grandmother has to make to prevent further disease progression of her ESRD. This is a sample of what kind of prevention?

  • Primary
  • Secondary
  • Tertiary
  • Quaternary

Which of the following is an example of tertiary prevention?

  • Implementing smoking ordinances in the city to prevent risk of lung cancer
  • Promoting use of seat belts to prevent car accidents
  • Screening of hearing and vision loss in the community center among old adults
  • Regular pain management for patient undergoing chemotherapy

A patient experienced a myocardial infarction four weeks ago and is currently participating in the daily cardiac rehabilitation sessions at the local fitness center. In what level of prevention is the patient participating?

  • Primary
  • Secondary
  • Tertiary
  • Quaternary

Situation

Proper communication and responses is essential in every nurse-patient relationship.

The nurse is caring for a client who is 4 days post-op for a transverse colostomy. The client is ready for discharge and asks the nurse to empty his colostomy pouch. What is the best response by the nurse?

  • You should be emptying the pouch yourself.
  • Let me demonstrate to you how to empty the pouch.
  • What have you learned about emptying your pouch?
  • Show me what you have learned about emptying your pouch.

A 3 year-old child has tympanostomy tubes in place. The child’s parent asks the nurse if he can swim in the family pool. The best response from the nurse is:

  • Your child should not swim at all while the tubes are in place.
  • Your child may swim in your own pool but not in a lake or ocean.
  • Your child may swim if he wears ear plugs.
  • Your child may swim anywhere.

The feeling of trust can best be established by the nurse during the process of the development of a nurse-client relationship by which of these characteristics?

  • Reliability and kindness
  • Demeanor and sincerity
  • Honesty and consistency
  • Sympathy and appreciativeness

A client, recovering from alcoholism, asks the nurse, What can I do when I start recognizing relapse triggers within myself?» How might the nurse best respond?

  • When you have the impulse to stop in a bar, contact a sober friend and talk with him.
  • Go to an AA meeting when you feel the urge to drink.
  • It is important to exercise daily and get involved in activities that will cause you not to think about drug use.
  • Let’s talk about possible options you have when you recognize relapse triggers in yourself.

One hour before the first treatment is scheduled, the client becomes anxious and states he does not wish to go through with electroconvulsive therapy. Which response by the nurse is most appropriate?

  • I’ll go with you and will be there with you during the treatment.
  • You’ll be asleep and won’t remember anything.
  • You have the right to change your mind. You seem anxious. Can we talk about it?
  • I’ll call the health care provider to notify them of their decision.

Situation

Infection control measures are done specifically in different types of conditions including proper management to treat diseases, and preventive actions.

Antiretroviral agents, such as AZT, are used in the management of AIDS. Which of the following is NOT an action expected of these drugs.

  • They prolong the life of the client with AIDS.
  • They reduce the risk of opportunistic infections
  • They shorten the period of communicability of the disease.
  • They are able to bring about a cure of the disease condition.

A barangay had an outbreak of German measles. To prevent congenital rubella, what is the BEST advice that you can give to women in the first trimester of pregnancy in the barangay?

  • Advice them on the signs of German measles.
  • Avoid crowded places, such as markets and moviehouses.
  • Consult at the health center where rubella vaccine may be given.
  • Consult a physician who may give them rubella immunoglobulin.

You were invited to be the resource person in a training class for food handlers. Which of the following would you emphasize regarding prevention of staphylococcal food poisoning?

  • All cooking and eating utensils must be thoroughly washed.
  • Food must be cooked properly to destroy staphylococcal microorganisms.
  • Food handlers and food servers must have a negative stool examination result.
  • Proper handwashing during food preparation is the best way of preventing the condition.

In a mothers’ class, you discussed childhood diseases such as chicken pox. Which of the following statements about chicken pox is correct?

  • The older one gets, the more susceptible he becomes to the complications of chicken pox.
  • A single attack of chicken pox will prevent future episodes, including conditions such as shingles.
  • To prevent an outbreak in the community, quarantine may be imposed by health authorities.
  • Chicken pox vaccine is best given when there is an impending outbreak in the community.

Complications to infectious parotitis (mumps) may be serious in which type of clients?

  • Pregnant women
  • Elderly clients
  • Young adult males
  • Young infants

The youngest child of the Yen family is vomiting every after feeding. She is then diagnosed with pyloric stenosis. This is classified as a:

  • Health threat
  • Health deficit
  • Foreseeable crisis
  • Stress point

Mr. and Mrs. Tanseco are discussing their financial budget for their son’s college fees. They are also planning to give him enough allowance for him to afford healthy meals, however this would entail the to cut back on some of their current expenses. They are experiencing what kind of problems?

  • Health threat
  • Health deficit
  • Foreseeable crisis
  • Stress point

Kiara recently gave birth and wants to commit to be a full-breastfeeding mother. However, she does not know what to do. You plan to teach her the key points of breastfeeding.

  • Wellness potential
  • Health deficit
  • Foreseeable crisis
  • Stress point

In a recent house inspection, Nurse Mario saw cockroaches and rodents running around in the kitchen. He is especially concerned as the rats were near the toddler’s snacks and formula milk. He raises this concern to the parents.

  • Health threat
  • Health deficit
  • Foreseeable crisis
  • Stress point

The COVID-19 pandemic led to the loss of numerous job opportunities and positions across different fields of specializations. Mr. Carlos lost his primary source of income as a door-to-door salesman since people are not allowed to leave their residences. What kind of problem is this?

  • Health threat
  • Health deficit
  • Foreseeable crisis
  • Stress point

This Law acknowledges that women who have retaliated against their partner or who commit violence as a form of self-defense may have suffered from Battered Woman Syndrome (BWS). Any victim who suffers from BWS should be diagnosed by a Psychiatric expert or a clinical psychologist.

  • RA 9262
  • RA 9288
  • RA 9482
  • RA 7719

RA 9262 is the law for Violence against Women and Children.

RA 9288 is the Newborn Screening Act of 2004. RA 9482 is the Anti-Rabies Act of 2007. RA 7719 is the Blood Services Act of 1994

The purpose of this law is to mandate compulsory basic immunization for infants and children below eight years of age in the Philippines. The law aims to promote the welfare of children and provide them with opportunities for a useful and happy life.

  • PD 651
  • PD 825
  • PD 856
  • PD 996

This law is a national policy that mandates the Philippine government to comprehensively address the needs of Filipino citizens when it comes to responsible parenthood and reproductive health.

  • RA 10028
  • RA 10121
  • RA 10152
  • RA 10354

The State commits itself to promoting the well-being of people by ensuring that: mental health is valued, promoted and protected; mental health conditions are treated and prevented; timely, affordable, high-quality, and culturally- appropriate mental health care is made available to the public. This law is the?

  • RA 10653
  • RA 10871
  • RA 10912
  • RA 11036

This act provides for the creation of a national program of air pollution management focusing primarily on pollution prevention; for the promotion of mass media communication in order to create social awareness and active participation in air quality planning and monitoring.

  • RA 8504
  • RA 8749
  • RA 8976
  • RA 9211

Situation

Each member of the profession adheres to a set of core ethical principles in nursing. The nursing code of ethics helps nurses make good judgment calls, provide high-quality care, and support patients and their loved ones emotionally, physically, and mentally.

Nurse Bea diligently performs her job. She attends to her patients’ needs, keeps them warm and comfortable, and maintains their safety at all times. Her actions help her to maintain the trust of her patients to her. She exhibits which of the following?

  • Fidelity
  • Justice
  • Beneficence
  • Veracity

The patient makes a decision that is against the medical advice of her physician. All advantages and disadvantages were mentioned, and still she chooses to go home against the medical advice. She is no inebriated or under influence of any substance. You advocated her decision, but still provided essential discharge instructions. This exhibits?

  • Fidelity
  • Justice
  • Autonomy
  • Veracity

Typhoon Karina hits the cities with strong winds, and rains – destructing the cities of Japan. Nurse Kate ensures that each families get relief packs during the rescue mission. She also prioritizes the people who are in need of immediate attention than those who are still able to walk and do their activities of daily living.

  • Fidelity
  • Justice
  • Autonomy
  • Veracity

When giving a high alert medication, nurse Paolo, despite his many years of experience, double checks the order and medication with another nurse. He exhibits which of the following?

  • Fidelity
  • Justice
  • Beneficence
  • Nonmaleficence

Nurse Enzo is prepping his patient before they go into surgery. Changes were made and the patient is to be sedated during the procedure. His patient suddenly exclaims, “I am scared as to how the anesthesia will affect me. Will I be asleep the whole time? How long will I be out?” He then calls for the anesthesiologist to have them explain the sedation plans of the procedure, and to secure the consent. His patient expresses their relief, and agrees to the plan. What does he exhibit?

  • Fidelity
  • Justice
  • Beneficence
  • Nonmaleficence

Situation

Programs of the DOH are implemented across the Philippines to help the Filipino achieve Universal health care

These are the key strategies of MNCHN, except:

  • Ensuring universal access to and utilization of MNCHN core package services and interventions directed not only to individual women of reproductive age and newborns at different stages of the life cycle.
  • Establishment of a service delivery network at only the primary level of care.
  • Organized use of instruments for health systems development
  • Rapid build-up of institutional capacities of DOH and PhilHealth.

Nurse Maloi advocates her clients to have their prenatal visits. What is the recommended minimum number of visits throughout the course of pregnancy?

  • 6
  • 8
  • 4
  • 5

Essential newborn care is also practiced in every delivery to ensure that all needs of the newborn are met, and protection is given. These are the interventions done within the first 90 minutes, except:

  • Immediate thorough drying
  • Skin-to-skin contact between mother and newborn
  • Early Cord clamping 1 minute after birth is recommended
  • Early initiation of breastfeeding (within 1 hour after birth)

The Family Planning program started in the 1970s as a family planning service delivery component to achieve fertility reductions. FP is means to prevent high-risk pregnancies brought about by the following conditions, except:

  • Being too young (less than 20 years old) or too old (over 33 years old)
  • Having had too many (4 or more) pregnancies
  • Having closely spaced (too close) pregnancies (less than 36 months)
  • Being too ill or unhealthy/too sick or having an existing disease or disorder like iron deficiency anemia.

Nurse Maloi also advocates for family planning fit according to the lifestyle and preference of the clients. She advocates the following barrier family planning methods, except:

  • Condom
  • Intrauterine device
  • Cervical cap
  • Diaphragms

Situation

The spread of an infection within a community is described as a “chain,” several interconnected steps that describe how a pathogen moves about. Infection control and contact tracing are meant to break the chain, preventing a pathogen from spreading.

This is any person, animal, arthropod, plant, soil, or substance (or combination of these) in which an causative agent normally lives and multiplies, on which it depends primarily for survival, and where it reproduces in such numbers that it can be transmitted to a susceptible host.

  • Pathogen/Causative agent
  • Portal of entry
  • Susceptible host
  • Reservoir

By encouraging rest and balance in our lives, you try to minimize risks in what part of the chain of infection?

  • Pathogen/Causative agent
  • Portal of entry
  • Susceptible host
  • Reservoir

You always change your gloves and practice hand hygiene before and after each patient. You minimize the risk in what part of the chain?

  • Mode of transmission
  • Portal of exit
  • Susceptible host
  • Reservoir

You make sure to collect all instruments every after surgery to have them sterilized. You know that this minimizes risk in?

  • Pathogen/Causative agent
  • Portal of entry
  • Susceptible host
  • Reservoir

You practice sterility in placing sterile dressings on surgical wounds. You know that this minimizes risk in?

  • Pathogen / Causative agent
  • Portal of entry
  • Susceptible host
  • Reservoir

Situation

Malaria is a life-threatening parasitic disease transmitted to humans through the bite of infected mosquitoes. The Department of Health on February 7, 2023 reported that all provinces in the country have achieved malaria-free status except for Palawan.

This agent causes the most serious type of malaria because of high parasitic densities in blood, and causes malignant tertian malaria. This is most common in the Philippines.

  • Plasmodium falciparum
  • Plasmodium vivax
  • Plasmodium ovale
  • Plasmodium malaria

Clinical manifestations of malaria in three stages. This stage happens in 4-6 hours with recurring high grade fever, severe headache, vomiting, abdominal pain, and blue face.

  • Cold Stage
  • Warm stage
  • Hot stage
  • Diaphoretic stage

Conducting the early diagnosis and prompt treatment for malaria is done to prevent further spread of the disease. This method is based on the examination of blood smear of the patient through a microscope.

  • Early diagnosis
  • Clinical method
  • Microscopic method
  • Chemoprophylaxis

Sustainable Preventive and Vector Control Measures refer to the adoption of measures for the prevention and control against the malaria parasite and the mosquito vector. This is the cutting of the vegetation overhanging along stream banks to expose the breeding stream to sunlight, rendering it unsuitable for mosquito vector habituation.

  • Insecticide-Treatment of Mosquito Net
  • House Spraying
  • On stream seeding
  • On stream clearing

Situation

The Integrated Management of Childhood Illness (IMCI) is an integrated approach to child health that focuses on the well-being of the whole child. IMCI aims to reduce death, illness and disability, and to promote improved growth and development among infants and children aged less than 5 years.

In Integrated Management of Childhood Illness, severe conditions generally require urgent referral to a hospital. Which of the following severe conditions DOES NOT always require urgent referral to a hospital?

  • Mastoiditis
  • Severe dehydration
  • Severe pneumonia
  • Severe febrile disease

A client was diagnosed as having Dengue fever. You will say that there is slow capillary refill when the color of the nailbed that you pressed does not return within how many seconds?

  • 3
  • 5
  • 8
  • 10

A 3-year old child was brought by his mother to the health center because of fever of 4-day duration. The child had a positive tourniquet test result. In the absence of other signs, which is the most appropriate measure that the PHN may carry out to prevent Dengue shock syndrome?

  • Insert an NGT and give fluids per NGT.
  • Instruct the mother to give the child Oresol.
  • Start the patient on intravenous fluids STAT.
  • Refer the client to the physician for appropriate management.

A 5-month old infant was brought by his mother to the health center because of diarrhea occurring 4 to 5 times a day. His skin goes back slowly after a skin pinch and his eyes are sunken. Using the IMCI guidelines, you will classify this infant in which category?

  • No signs of dehydration
  • Some dehydration
  • Severe dehydration
  • The data is insufficient.

Based on assessment, you classified a 3-month old infant with the chief complaint of diarrhea in the category of SOME DEHYDRATION. Based on IMCI management guidelines, which of the following will you do?

  • Bring the infant to the nearest facility where IV fluids can be given.
  • Supervise the mother in giving 200 to 400 mL of Oresol in 4 hours.
  • Give the infant’s mother instructions on home management.
  • Keep the infant in your health center for close observation.

A mother is using Oresol in the management of diarrhea of her 3-year old child. She asked you what to do if her child vomits. You will tell her to:

  • Bring the child to the nearest hospital for further assessment.
  • Bring the child to the health center for intravenous fluid therapy.
  • Bring the child to the health center for assessment by the physician.
  • Let the child rest for 10 minutes then continue giving Oresol more slowly.

Situation

Nurse Ysabel reviews the fundamentals as she studies for the board exam.

What is the order of the nursing process?

  • Assessing, diagnosing, implementing, evaluating, planning
  • Diagnosing, assessing, planning, implementing, evaluating
  • Assessing, diagnosing, planning, implementing, evaluating
  • Planning, evaluating, diagnosing, assessing, implementing

A client is receiving 115 ml/hr of continuous IVF. Nurse Ysabel notices that the venipuncture site is red and swollen. Which of the following interventions would she perform first?

  • Stop the infusion
  • Call the attending physician
  • Slow that infusion to 20 ml/hr
  • Place a cold towel on the site

She will now proceed to give the appropriate feeding for her patient. Which of the following is inappropriate nursing action when administering NGT feeding?

  • Place the feeding 20 inches above the point of insertion of NGT
  • Introduce the feeding slowly
  • Instill 60 mL of water into the NGT after feeding
  • Assist the patient in fowler’s position

During application of medication into the ear, which of the following is inappropriate nursing action?

  • In an adult, pull the pinna upward
  • Instill the medication directly into the tympanic membrane
  • Warm the medication at room or body temperature
  • Press the tragus of the ear a few times to assist flow of medication into the ear canal

Postural drainage to relieve respiratory congestion should take place:

  • Before meals
  • After meals
  • At the nurse’s convenience
  • At the patient’s convenience

Recalls II

Situation

Nurse Kaye is appointed as the nurse supervisor in a municipal health unit. She is guided by the managerial functions to properly guide her team.

In the management process, Nurse Kaye aims to forecast and project what is most likely to occur in the future. She knows that this encompasses which phase of the management process

  • Planning
  • Organizing
  • Directing
  • Controlling

To promote the growth of her team, Nurse Kaye utilizes performance appraisal as a tool to determine actual performance compared with the desired output. Which of the following phases does this action fall into

  • Planning
  • Organizing
  • Directing
  • Controlling

As a nurse supervisor, Nurse Kaye must possess which type of supervisory skill in order to understand and work with other people by fostering harmonious relationships among the group

  • Technical skills
  • Human relations skills
  • Administrative skills
  • Decision making skills

In order to critically analyze information and problems and make appropriate decisions, Nurse Kaye must possess which supervisory skill

  • Technical skills
  • Human relations skills
  • Administrative skills
  • Decision making skills

To function harmoniously in the attainment of organizational objectives, Nurse Kaye assembles a team designated for health programs such as Family Planning Program. Which principle of organization is being applied in this case

  • Span of control
  • Unity of command
  • Coordination
  • Division of work

Nurse Joei is part of a team assigned in the control of communicable diseases, he knows which role and function of Department of Health (DOH)is involved in health emergency response services and in cases of epidemic and other widespread public danger

  • Leadership in Health
  • Enabler and Capacity Builder
  • Administrator of Specific Services
  • Health Sector Reform Agenda

Nurse Joie knows that the overriding goal of Department of Health (DOH) is:

  • FOURmula ONE for Health
  • Health Sector Reform Agenda
  • National Objectives for Health
  • Inter Local Health System

A sector-wide approach to implement critical interventions in the implementation of HSRA is delineated in its framework. Nurse Joei has kept in mind that the framework for the implementation of Health Sector Reform Agenda (HSRA) is:

  • Primary Health Care
  • Inter Local Health System
  • National Objectives for Health
  • FOURmula ONE for Health

FOURmula ONE for Health is the overarching philosophy to achieve end goals. Nurse Joei is aware that all but one are goals of FOURmula ONE for Health?

  • Better health outcomes
  • More responsive health systems
  • Equitable health care financing
  • Good governance

The devolution of health services from the national level to Local Government Unit (LGUs) was emphasized in the Local Government Code. Nurse Joei is aware that this is the focal point of:

  • RA 9173
  • EO 102
  • RA 7160
  • LOI 949

Nurse Shape focuses on the people as the center of any health and development effort. Which of the following should he list down as key ingredients to successful and sustainable community-based approach/health initiatives? Select all that apply.

  • Active community participation
  • Multi-sectoral collaboration
  • Strong support and guidance from local governments and technical experts
  • Involvement of community leaders, committees and other groups

As a public health nurse, Nurse Shape assumes various roles and responsibilities in the prevention and control of NCDs. Which role does he perform when is concerned with promoting health as well as reducing behavior-induced disease.

  • Health Advocate
  • Health Educator
  • Community Organizer
  • Health Trainer

Nurse Shape conducts hypertension screening among the residents of the community. He is aware that this is under which level of prevention?

  • Primary Prevention
  • Secondary Prevention
  • Tertiary Prevention
  • Health Promotion

Alice who is diagnosed with DM Type II was given advice by Nurse Shape to identify and prevent complications related to DM. This is considered as?

  • Primary Prevention
  • Secondary Prevention
  • Tertiary Prevention
  • Health Promotion

Nurse Shape encourages parents to bring their children to the health center for their immunization. He is performing which level of prevention?

  • Primary Prevention
  • Secondary Prevention
  • Tertiary Prevention
  • Health Promotion

According to the World Health Organization (WHO), this is the presumptive identification of unrecognized disease or defect by the application of tests, examination or other procedures which can be applied rapidly. Nurse Jerico understands that this pertains to which of the following:

  • Risk Factor
  • Community Diagnosis
  • Screening
  • Triangulation

Jona, 43 years old, arrived at the clinic for consultation. She has a BMI of 26 with a waist circumference of 95 cm and hip circumference of 105. Nurse Jerico assesses her degree of risk of comorbidity based on waist-to-hip ratio (WHR). All but one are incorrect when interpreting the waist-to-hip (WHR) ratio of the client

  • Jona has a WHR of 0.85 which is considered normal.
  • Jona has a WHR of 1.0 which is considered normal.
  • Jona has a WHR of 0.90 which is considered as android or central obesity.
  • Jona has a WHR of 1.12 which is considered as android or central obesity.

Nurse Jerico knows that the most accurate and reliable technique for indirect BP measurement is:

  • Palpatory method using a mercury manometer
  • Palpatory method using aneroid BP
  • Auscultatory method using a mercury manometer
  • Auscultatory method using aneroid BP

The municipal health center offers free screening for elevated cholesterol. As part of the committee, Nurse Jerico discusses the guidelines and he is aware that the normal cholesterol level should be:

  • <150 mg/50 mL
  • <200 mg/100 mL
  • 200 to 239 mg/100 mL
  • 240 mg/100 mL and above

Early detection and prompt treatment are integral to the management of cancer. Nurse Jerico provides health education to families at risk by including all of the following as warning signs of cancer, except?

  • Change in bowel habits
  • Sudden weight gain
  • Unexplained anemia
  • Dysphagia

Situation

As a community health nurse assisting a family in the care of their 83-year old grandfather, Hector, with Alzheimer’s disease. Nurse Whayee assesses the family’s use of safety precautions during a home visit.

Nurse Whayee performs home hazard appraisal for the family. None of the following suggest an unsafe condition leading to falls or injury, EXCEPT

  • Presence of insecure handrails
  • Presence of grab bars around tubs and showers
  • Adequacy of lighting at night
  • Anchored rugs and mats

Members of the family have opened up about their experiences in taking care of Hector. Nurse Whayee observes the family during the visit. She should recognize the following as signs of caregiver role strain, except? Select all that apply.

  • Feelings of anger and depression
  • Reports of increased physical energy and sufficient time for caregiving
  • Caregiving responsibilities does not interfere with other roles
  • Anxiety about ability to meet future care needs of the client
  • Difficulty performing routine tasks for the client

Nurse Whayee demonstrates infection control and prevention measures to the family. As a nurse, she knows that her major role in infection prevention is:

  • Provision of basic hand hygiene kits
  • Addressing challenges to infection prevention such as lack of running water for handwashing
  • Providing health teaching to client and caregivers
  • None of the above

Given the condition of Hector, Nurse Whayee was informed that the family was having a hard time with the elderly who is noncompliant to his medication regimen. Nurse Whayee knows that the Joint Commission International (JCI) abides by their patient safety goals.Which of the following should she initially perform in order to promote safety of using medications?

  • Compare the medication information the patient is currently taking with the medications ordered for the patient in order to identify and resolve discrepancies.
  • Obtain information on the medications the patient currently takes.
  • Define the types of medication information (dose, route, frequency, purpose) to be collected in different settings and patient circumstances.
  • Explain the importance of managing medication information to the patient

Nurse Whayee explores and supports the client’s choices while also considering viable options. The client discussed her plan to have a living will. In order for the client to make informed decisions, Nurse Whayee is aware that advocacy begins when?

  • First visit
  • Two days later
  • A week after
  • During termination of interaction

For the past 5 years, Nurse Earl has not been practicing the profession. He has been preoccupied with his pares overload business. He plans to return to work as a staff nurse. In order to do this, he must undergo training of:

  • 1 month practicum and 3 months didactic
  • 1 month didactic and 3 months practicum
  • 3 months practicum and 6 months didactic
  • 3 months didactic and 6 months practicum

Along with his desire to return to the hospital, Nurse Earl also plans to advance his studies further by enrolling in graduate programs in order to qualify as a clinical instructor in his alma mater. He is guided by the qualifications of a faculty in the college of nursing which requires him to comply with the following. Select all that apply.

  • Be a registered nurse in the Philippines
  • Have atleast one year of clinical practice in a field of specialization
  • Be a member of good standing in the accredited professional organization of nurses
  • Be a holder of masters degree in nursing, education or other allied medical and health sciences

Nurse Earl is aware that the professional regulatory board of nursing (PRBON) is composed of a chairperson and members who shall represent different areas of nursing which includes all of the following, except:

  • Nursing education
  • Leadership and management
  • Nursing service
  • Community health nursing

Nurse Earl understands that the conduct of the licensure examination for nurses in the country shall be in accordance with the PRC Modernization Act of 2000 also known as:

  • R.A. 7164
  • R.A. 8981
  • R.A. 7160
  • R.A. 877

Nurse Earl will renew his professional identification card. He is certain of which of the following that are included in the ID:

  • Full name of the registrant, serial number, signature of the Chairperson and Members of the Board
  • Full name of the registrant, date of registration, license number and date of issuance
  • Full name of the registrant, duly signed by the Chairperson of the Commission, bearing the date of registration, license number and date of issuance and expiration
  • Full name of the registrant, serial number, date of registration, license number and date of issuance and expiration and signature of the Chairperson and Members of the Board

Nurse Nycah intends to find out the most frequently occurring cause of contaminated water for the last six months as basis for water safety program. What measure of central tendency will she use to correctly do this?

  • Mean
  • Median
  • Mode
  • Frequency distribution

Nurse Nycah supervises a study on the compliance of barangay healthcare workers (BHWs) on infection control measures. Given that the subjects are aware that they are being observed on their compliance to hand hygiene, they are more likely to comply and perform such measures. This is referred to as:

  • Halo effect
  • Hawthorne effect
  • Error of leniency
  • Error of severit

As participants of the study, the barangay healthcare workers (BHWs) know that they are being observed as part of a study on infection control and prevention. This type of observation is commonly referred to as:

  • Overt observation
  • Covert observation
  • Field notes
  • Deception

A study on the perception of local residents on the emergence of Dengue cases in relation with poor environmental sanitation is being conducted. Nurse Nycah has reached a point where no new information is obtained and redundancy is achieved which pertains to:

  • Bracketing
  • Triangulation
  • Data Saturation
  • None of the above

Nurse Nycah will conduct a survey among the residents of the community. The eligibility criteria for the study will include the age of the respondent. She knows that age falls on what level of measurement?

  • Nominal
  • Ordinal
  • Interval
  • Ratio

The quality of the ethical nurse revolves around the ability to explain the rationale behind every action and recognize standards to which he or she will be held. The hallmark of nursing accountability is:

  • Quality and safety education for nurses
  • Accurate documentation
  • Compliance to standards of care
  • Ethical reasoning

As advocate of patients, Nurse Shel must take appropriate steps to safeguard their rights and privileges which includes observing the following guidelines. Select all that apply.

  • Respect the “Patients’ Bill of Rights” in the delivery of nursing care
  • Provide the patients or their families with all pertinent information including those which may be deemed harmful to their well-being
  • Uphold the patients’ rights when conflict arises regarding management of their care

According to the Code of Ethics, these are the goals of nursing practice

  • Accurate documentation
  • Quality and excellence in the care of patients
  • Individuality and totality of patients
  • Safeguarding the rights and privileges of patients

Nurses must know the definition and scope of nursing practice which are in the provisions of Philippine Nursing Act and Code of Ethics. The principal basis of the Code of Ethics is:

  • PRC Modernization Act
  • Patients’ Bill of Rights
  • Philippine Nurses Association
  • Code of Good Governance

The fifth-fold responsibility of a nurse pertains to the:

  • Promotion of health
  • Promotion of a spiritual environment
  • Alleviation of suffering
  • Restoration of health

In order to understand the behavior of the family as a dynamic, functioning unit, the Structural-Functional framework specifies four structural dimensions which includes all of the following except?

  • Role structure
  • Power structure
  • Reproductive function
  • Communication processes

Nurse Kayzee assesses the Guo family. In her data collection, which of the following should she include in the first-level assessment? Select all that apply.

  • Family structure, characteristics and dynamics
  • Socio-economic and cultural characteristics
  • Home and environment
  • Health status of each member
  • Values and practices on health promotion and disease prevention

As part of the first-level assessment, Nurse Kayzee identifies the lack of immunization of a family member as a:

  • Wellness Condition
  • Health Threat
  • Health Deficit
  • Foreseeable Crisis

Statements of the family nursing problems are reflected in the second-level assessment. All of the following are necessary health tasks for the family as a functioning unit except:

  • Recognize the presence of wellness state or health condition/problem
  • Utilize community resources for health care
  • Maintain a home environment inconducive to health maintenance and personal development
  • Provide nursing care to the sick, disabled, dependent or at-risk members

Nurse Kayzee assesses the home and environment of the Guo family. All of the following should be included in her assessment except:

  • Housing
  • Kind of neighborhood
  • Anthropometric data
  • Communication and transportation facilities available

Donna complaints of a cough that began three weeks ago. She has had fever, back pains, blood-streaked mucus and body malaise. She went to the health center for consultation. Nurse Arjay is aware that the primary diagnostic tool in the National Tuberculosis Program case finding is:

  • PPD
  • DSSM
  • X-ray
  • Immunochromatographic Test (ICT)

Nurse Arjay knows that the only contraindication for sputum collection is:

  • Weight loss
  • Syncope
  • Angioedema
  • Hemoptysis

All of the following are true about DSSM except?

  • Pulmonary TB symptomatics shall be asked to undergo other diagnostic tests (X-ray and culture), if necessary, only after they have undergone DSSM for diagnosis with three sputum specimens yielding negative results.
  • Barangay health workers in far flung areas are not allowed to do smearing and fixing of specimens.
  • Trained medical technologists or microscopists shall perform DSSM.
  • DSSM can be requested after the control of hemoptysis

In administering intradermal injection to a child, which of the following considerations should Nurse Arjay emphasize

  • Make sure the child understands that the injection is a punishment
  • The child should be heavily restrained during the procedure
  • Ask the child not to rub or scratch the injection site
  • Ask the parent to return for evaluation of the condition of the site after 72 hours

Donna underwent the Mantoux test as a screening for TB. Nurse Arjay knows that the test result will be read:

  • 24 hours after injection
  • 48 to 72 hours after injection
  • 24 to 72 hours after injection
  • After a week

Terrence is certain of the clinical manifestations of mumps. Nurse Yvette is correct when she identifies the following signs and symptoms for mumps. Select all that apply.

  • Fever and malaise
  • Anorexia
  • Orchitis
  • Painless swelling of the ears, jaws and neck

Terrence reports discomfort on his left testicle. Upon inspection, there is visible swelling. Nurse Yvette knows that in order to minimize edema and atrophy, she will instruct the client to do the following except:

  • The scrotum should be supported by a properly fitted suspensory, pillow or a sling between the thighs
  • Use soft packing between the scrotum and the support
  • Place a pillow or adhesive bridge between the thighs
  • Let the client lie on his left side

The client had inquiries regarding his diet. Nurse Yvette is correct when she provides which home instruction to Terrence?

  • Regular diet as tolerated
  • Soft or liquid diet as tolerated
  • Fruit juice and sour foods should be part of his diet
  • There are no dietary restrictions for the client

Nurse Yvette is aware that the most common complication of mumps in male adults is:

  • Encephalitis
  • Meningitis
  • Orchitis
  • Nuchal rigidity

Encephalitis may complicate mumps. Which signs and symptoms must the client watch out for in this case? Select all that apply.

  • Sudden rise in temperature
  • Stiff neck
  • Headache
  • Nausea and vomiting
  • Delirium
  • Diplopia

Nurse JD plans to resolve problems in scheduling of the staff in the community health center. In using the DECIDE Model as a tool for decision making, Nurse JD is aware that the initial step of this model is to:

  • Develop and implement a plan of action
  • Establish criteria
  • Define the problem
  • Identify the best alternative

Problem solving consists of the act of identifying a problem and implementing an active systematic process to solve problems. When faced with difficulties in the staffing, Nurse JD capitalizes on the positive characteristics to bring about a culture change by engaging staff in a healthy exchange of knowledge. This is also known as?

  • Shared decision making
  • Appreciative inquiry
  • SWOT analysis
  • DECIDE Model

Nurse JD implements the use of SOAPIE as the new format for charting in the progress notes. None of the following are subject data except:

  • X-ray result
  • Vital signs
  • Client’s perception of the problem
  • Laboratory results

As a change agent, Nurse JD plans on strategies to manage staff who are resistant to change. Which strategy assumes that staff members are essentially self-interested and providing information will assist staff in changing behavior to adopt change?

  • Normative-reeducative strategy
  • Empirical-rational strategy
  • Power-coercive strategy
  • None of the above

When faced with conflict within the team, Nurse JD has a tendency to identify shared goals and commits to working together to achieve resolution. This conflict management strategy is:

  • Accommodating
  • Compromising
  • Competing
  • Collaborating

To enhance communication among healthcare professionals, Nurse Geo ensures that the patient information is current and available for the meeting and offers the team insight into care issues with the goal of advocating for the patient. This is particular to which communication strategy?

  • Interprofessional Team Rounding
  • TeamSTEPPS
  • SBAR
  • Handover

When utilizing the SBAR communication tool, Nurse Geo is aware that R stands for:

  • Response
  • Reminder
  • Record
  • Recommendation

Transition in care when a patient is transferred from one unit to another or during a change of shift is referred to as:

  • Two-challenge rule
  • Call out
  • Check-back
  • Handover

When communicating with a widow who recently lost her husband due to a car accident, Nurse Geo must alter spoken messages in accordance with behavioral cues from the client. What characteristic does Nurse Geo consider?

  • Adaptability
  • Credibility
  • Timing and Relevance
  • Clarity and Brevity

While interviewing Jelly, Nurse Geo notices that Jelly is becoming more tense as he was about to give her medications. Which of the following is correct about Nurse Geo’s observations?

  • The patient is feeling anxious
  • The patient is depressed
  • The behavior suggests feeling of well-being
  • The patient is uncomfortable

In doing quick neurologic assessment, Nurse Anna utilizes the AVPU method. Upon assessment, she will classify the patient as unresponsive in which of the following conditions

  • The patient is alert and conscious
  • The patient responds only to verbal stimuli
  • The patient responds only to painful stimuli
  • The patient does not respond to all stimuli

When providing health teaching on the prevention of heat-induced illness, which of the following is inappropriate for Nurse Anna to include

  • Monitor fluid losses and weight loss during workout or exercise and replace fluids and electrolytes
  • Maintain adequate fluid intake and wear loose clothing
  • Plan outdoor activities between 10 AM to 2 PM
  • Avoid immediate re-exposure to high temperatures

Due to the extreme heat-index in some areas of the country, heat stroke can occur. Kim, one of the trainees for ROTC, presents with a temperature of 41 C with hot, dry skin and anhidrosis. She reportedly collapsed and has been delirious.Rapid temperature reduction was initiated within an hour. Which methods should Nurse Geo exclude as a cooling procedure?

  • Place cool sheets or cooling blankets on the patient
  • Apply ice to the neck, groin, chest and axilla while spraying tepid water
  • Immerse the patient in a cold water bath
  • Administration of sodium bicarbonate

Which of the following is considered as an active external rewarming method?

  • Forced-air warming blankets
  • Cardiopulmonary bypass
  • Peritoneal lavage
  • Warmed humidified oxygen by ventilator

Nurse Geo assesses a family who is suspected to have experienced food poisoning. He aims to determine the source and type of food poisoning. Which of the following options pertains to a serious form of food poisoning?

  • Anthrax
  • Botulism
  • Smallpox
  • Tularemia

Situation

Attending to public health needs is of utmost concern to Nurse Corr. Bringing attention to health promotion and disease prevention through various programs in the community can help decrease health disparities. The following questions apply.

Nurse Corr initiates a community-based program intended to reduce incidence of teenage smoking. She influences the youth to make good choices about habits and practices that are detrimental to health. As a nurse educator, Nurse Corr knows that the prevalent compound in cigarettes deposited in lung passages that paralyzes the cleaning mechanisms (cilia) and damages the air sacs (alveoli) is

  • Nicotine
  • Carbon monoxide
  • Tar
  • Ethanol

Nurse Corr recalls that the Third International Conference on Health Promotion was held in 1991 where?

  • Ottawa, Canada
  • Sundsvall, Sweden
  • Adelaide, Australia
  • Alma Alta, Russia

The Ottawa Charter for Health Promotion states that health promotion must occur on these fronts. Select all that apply.

  • Building healthy public policy
  • Creating environment which support healthy living
  • Strengthening community action
  • Helping people develop their skills
  • Reorienting health system

Nurse Corr aims to achieve a sustainable community health development grounded on health promotion. Which of the following approaches strikes at the root of the problem and addresses the social determinants of health?

  • Community-based approach
  • Integrated approach
  • Comprehensive approach
  • Community organizing

Nurse Corr understands that in order to operationalize the concept of health promotion, the Ottawa Charter recommended the five action areas for health promotion which includes the following. Select all that apply.

  • Build healthy public policy
  • Create supportive environments
  • Strengthen community action
  • Develop personal skills
  • Reorient health services

Situation

Nurse Leslie was appointed as the team leader in a rural health unit. She immerses herself in the lives of the residents. Through her adviser role in the conduction of COPAR, she aims to have a self-sustained community fully equipped to manage problems on their own.

As a public health nurse, Nurse Leslie knows that the primary goal of COPAR is to:

  • To enhance community empowerment and self-reliance
  • To encourage the residents to identify existing problems and pressing issues in the community
  • To help the community allocate funds proportionately
  • To assist the community in prioritizing problems

Nurse Leslie remembers the vital role of the community in COPAR. She knows that they are considered as:

  • Financer
  • Beneficiaries
  • Subjects
  • Co-researchers

Which of the following pertains to the process whereby the community members develop capability to assess their health needs and problems, plan and implement actions and sustain organizational structure to support health initiatives by the people?

  • Community Participation
  • Community Diagnosis
  • Community Organizing
  • Community Profiling

Nurse Leslie understands that during this phase in COPAR, the community assumes responsibility in managing their health needs as the nurse gradually prepares for turn-over of work and develops plan for monitoring and subsequent follow-up with the organizations activities. Which phase does this pertain to?

  • Training
  • Pre-entry
  • Entry
  • Phase-out

Which of the following is the initial step in community diagnosis?

  • Determining the data to be collected
  • Defining the study population
  • Determining the objectives
  • Developing the instrument

Situation

Nurse Mel supervises the immunization program of the clinic by teaching BHWs and midwives the proper storage, administration and education regarding vaccines.

When teaching about the principles of the Expanded Program on Immunization. All are incorrect about the principles of EPI, except

  • It is safe and immunologically effective to administer all EPI vaccines on the same day at the same site of the body
  • Vaccines should be restarted from the beginning even if the interval between doses exceeded the recommended interval by months or years
  • Repeat BCG vaccination if the child develops a scar after the first injection
  • Giving doses of a vaccine at less than the recommended 4 weeks interval may lessen the antibody response

Nurse Mel must teach that which of the following vaccines must be wrapped in foil and protected from sunlight

  • BCG
  • Hep B
  • Measles
  • Tetanus Toxoid

Nurse Mel knows that the component of DPT vaccine containing a killed bacteria is:

  • Diphtheria
  • Pertussis
  • Tetanus Toxoid
  • Hep B

Nurse Mel adheres to the appropriate temperature monitoring of vaccines in the clinic. He understands that the temperature monitoring of the cold chain should be done in how many times and with what interval?

  • Once daily in the morning
  • Once daily in the afternoon
  • Twice a day early in the morning and in the afternoon
  • Thrice daily every 4 hours

Completing the five doses of Tetanus Toxoid for mothers provides lifetime immunity. Which of the following gives protection to infants from neonatal tetanus and 5 years protection for the mother?

  • TT1
  • TT2
  • TT3
  • TT4

Nurse Andre is aware of the components of FHSIS. He emphasizes the fundamental block of FHSIS is the:

  • Target/client list
  • Tally report
  • Output report
  • Family treatment record

To properly monitor pregnant clients registered to receive prenatal care, which component will Nurse Andre use?

  • Target/client lists
  • Tally report
  • Output report
  • Family treatment record

In FHSIS, the ideal reporting of neonatal and maternal deaths should be done:

  • Weekly
  • Monthly
  • Quarterly
  • Upon occurrence of event

Nurse Andre knows that which level of the DOH management produces the output report:

  • Regional Health Office
  • Rural Health Unit
  • Barangay Health Center
  • Public Health Office

The lowest level of reporting unit expected to report health services is the:

  • Barangay Health Station
  • Regional Health Office
  • Public Health Office
  • Department of Health

Situation

Daisy, the community health nurse, developed an advocacy leaning towards mental health promotion and increasing awareness among the population on mental health and psychosocial status.

As an advocate of mental health, she understands the four facets presented as a health burden. Which burden pertains to the stigma associated with all forms of mental health illness?

  • Defined burden
  • Undefined burden
  • Hidden burden
  • Future burden

Which burden results from the aging of the population, increasing social problems and unrest inherited from an existing burden?

  • Defined burden
  • Undefined burden
  • Hidden burden
  • Future burden

Betty has been recently diagnosed with bipolar disorder. Her sudden mood and behavioral changes had a devastating impact to her work and responsibilities as an adult. Which burden pertains to this situation?

  • Defined burden
  • Undefined burden
  • Hidden burden
  • Future burden

Because of her mental health disorder, her family has been very worried about Betty’s wellbeing. Her struggles to make ends meet given her mental health issue has forced her family to intervene and suggested for her to voluntarily admit herself in a psychiatric unit for observation. Which burden is being described in this scenario?

  • Defined burden
  • Undefined burden
  • Hidden burden
  • Future burden

To grasp the community’s overall mental health status, Nurse Daisy determined the incidence and prevalence of mental illnesses. She is fully aware that incidence rate pertains to the:

  • Number of new cases of mental illness over a specified period of time in the population at risk
  • Number of new and old cases of mental illness in the total population
  • Risk of exposure of persons in a given year
  • Number of registered deaths from a specific disease over the number or registered cases from same specific disease in the same year

Situation

Nurse Han will conduct a study examining the effect of regular elastic band exercises on the production of free radicals and antioxidant enzymes in older adults. The following questions apply.

In this particular study, the independent variable is the:

  • Production of free radicals and antioxidant enzymes
  • Regular elastic band exercises
  • Free radicals
  • Antioxidants

Nurse Han knows that “production of free radicals and antioxidant enzymes” is considered as the:

  • Independent variable
  • Dependent variable
  • Manipulation
  • Control

When asked about the definition of HbA1c, which of the following should Nurse Han conclude as its conceptual definition:

  • The normal range of glycosylated hemoglobin range from 4% to 6%
  • HbA1c is a laboratory value that reflects the average blood sugar reading for a patient over 90 days
  • HbA1c are byproducts of fat breakdown that accumulate in the blood and urine
  • HbA1c is a biomedical measure of an individual’s success in managing their type 1 diabetes

The operational definition of HbA1c should be:

  • The normal range of glycosylated hemoglobin range from 4% to 6%
  • HbA1c is a laboratory value that reflects the average blood sugar reading for a patient over 90 days
  • HbA1c are byproducts of fat breakdown that accumulate in the blood and urine
  • HbA1c is a biomedical measure of an individual’s success in managing their type 1 diabetes

This part of a research refers to the clear concise statement of the specific goal or study. Which of the following would Nurse Han pertain to in relation to this

  • Research problem
  • Research objective
  • Research purpose
  • Research question

Recalls III

Situation

On 19 September 2019, an outbreak of polio was declared in the Philippines. Children in the Philippines are at risk of lifelong paralysis because of this outbreak.

What is the mode of transmission of poliomyelitis?

  • Blood-borne transmission
  • Fecal- oral route
  • Vector-borne transmission
  • Airborne droplets

What is the route of administration of OPV?

  • Oral
  • Intradermal
  • Intramuscular
  • Subcutaneous

What is the appropriate temperature to store oral polio vaccine?

  • -15°C to -25°C
  • +2°C to +8°C
  • -2°C to -8°C
  • +15°C to +25°C

Along with the oral polio vaccine which of the following vaccines is also one of the most sensitive to heat?

  • DPT/ Hep B
  • BCG
  • Tetanus Toxoid
  • Measles

The following are the general principles which apply in vaccinating children. Select all that apply.

  • It is safe and immunologically effective to administer all EPI vaccines on the same day at different sites of the body
  • The vaccination schedule should be restarted from the beginning if the interval between doses exceeds the recommended interval by months or years.
  • Moderate fever, malnutrition, mild respiratory infection, cough, diarrhea, and vomiting are not contraindications to vaccination.
  • Giving doses of a vaccine at less than the recommended 4-week interval may lessen the antibody response.
  • Lengthening the interval between doses of vaccines leads to lesser antibody levels

Situation

Community Health Nurse Elsa is providing health teaching to Brgy. Macamot regarding the Patterns of occurrence and distribution of diseases. The following questions are discussed.

This pertains to the intermittent occurrence of a few isolated and unrelated cases in a given locality

  • Sporadic
  • Epidemic
  • Endemic
  • Pandemic

This is the continuous occurrence throughout a period of time, of the usual number of cases in given locality

  • Sporadic
  • Epidemic
  • Endemic
  • Pandemic

Which of these involves an unusually large number of cases in a relatively short period of time?

  • Sporadic
  • Epidemic
  • Endemic
  • Pandemic

This is the simultaneous occurrence of an epidemic of the same disease in several countries

  • Sporadic
  • Epidemic
  • Endemic
  • Pandemic

Rabies is an example of a:

  • Sporadic
  • Epidemic
  • Endemic
  • Pandemic

Situation

Patient Nora arrived at the Seattle Grace Hospital with complaints of fever, profuse sweating, malaise, and recurrent chills. Upon assessment, she also reported a recent vacation in Palawan. Nurse Grace suspects Malaria, further assessment is performed.

Patient Nora asked Nurse Grace, what is the vector of Malaria?

  • Oncomelania quadrasi
  • Female Anopheles
  • Aedes poecilius
  • Aedes aegypti

Upon health teaching, Nurse Grace includes measures for the prevention and control of the malaria parasite and the mosquito vector. Which of the following should not be included

  • Construction of bio-ponds for larvivorous fish propagation.
  • Cutting of vegetation overhanging along stream banks to expose breeding stream to sunlight.
  • Avoid outdoor night activities, particularly during the vectors’ peak biting hours from 9 pm-3 pm.
  • Changing water and scrubbing sides of lower vases once a week.

There are various species of plasmodium, among the choices which of the following is found to be common in the Philippines

  • P. ovale
  • P. malariae
  • P. vivax
  • P. falciparum

Which blood cells are affected by the Malaria parasite?

  • Red blood cells
  • White blood cells
  • Platelets
  • All of the above

Which of the following is not a clinical manifestation of Malaria?

  • Fatigue
  • Petechiae
  • Pale skin
  • Cold extremities

Situation

Nurse Sara is a newly employed community health nurse. She is advocating the use of the ten herbal plants.

The herbal plant used to expel round worms ascariasis is:

  • Akapulko
  • Ampalaya
  • Niyog Niyogan
  • Tsaang Gubat

The herbal medicine appropriate for kidney stones is:

  • Sambong
  • Tsaang gubat
  • Niyog Niyogan
  • Bayabas

“Botika ng Barangay” (BnB) refers to a drug outlet to sell, distribute, offer for sale, and or make available low-priced generic home remedies over the counter drugs. Which one is not included?

  • Cotrimoxazole
  • Amoxicillin
  • Metformin
  • Aspirin

Public Health Nurse Kiara is assigned to pediatric clients in the barangay to administer medications and vaccines. As a registered nurse she should be knowledgeable on how to provide the proper administration. When administering vaccines, the first step Nurse Kiara should do is to:

  • Reconstitute using diluent supplied.
  • Check the content prior to drawing up.
  • Select the proper needle size.
  • Check the vial for the expiration date.

After administering the vaccine, which of the following Nurse Kiara should not include in the documentation

  • Date
  • Lot number
  • Manufacturer
  • Needle Gauge

Situation

A recent Nurse licensure examination passer Nurse Kelsey took the opportunity to work in a rural health unit in the Municipality of Bombon. She reviews the concepts and principles in Primary health care.

The global agency that initiated the Alma Ata Cconference on Primary Health is the:

  • World Health Organization (WHO)
  • United Nations Children’s Fund (UNICEF)
  • Department of Health
  • Centers for Disease Control and Prevention (CDC)

Which of the following is not included in the four cornerstones/pillars of Primary Health Care?

  • Active community participation
  • Intra and Intersectoral linkages
  • Use of the latest technology
  • Support mechanism made available

Nurse Kelsey is correct in enumerating the members of the village or barangay health workers if she answers:

  • Health auxiliary volunteers, traditional birth attendants, and healers.
  • Health auxiliary volunteers, traditional birth attendants, and midwives.
  • Health auxiliary volunteers, public health nurses, and midwives.
  • Public health nurses and midwives only.

Nurse Kelsey recognizes that health services offered are to be in accordance with the prevailing beliefs and practices of the intended clients of care. This primary health care key principle is known as:

  • Availability
  • Acceptability
  • Affordability
  • Accessibility

When the hospital reaches out to the Department of Social Welfare and Development (DSWD) this linkage is an example of:

  • Network linkages
  • Inter-sectoral linkages
  • Intra-sectoral linkages
  • Multiple linkages

Situation

Patient Mina went to the outpatient department. She reported 3 weeks of cough, coughing up blood, and fatigue. She also noticed that she lost 6 kilograms within 1 month, and excessive sweating particularly at night. The doctor ordered several tests and confirmed the diagnosis of Pulmonary Tuberculosis.

Patient Mina is concerned about how she acquired pulmonary tuberculosis. How should Nurse Glenda respond?

  • It is acquired through ingestion of contaminated foods.
  • It is acquired through inhalation of bacteria which travel in droplet nuclei that become aerosolized.
  • It is acquired through the use of infected needles.
  • It is acquired through a bite of a mosquito carrying the bacteria.

Patient Mina is asking, “Why did the doctor order a chest x-ray?”. Nurse Glenda should answer:

  • To confirm the diagnosis of PTB.
  • To know the extent of the lesion in the lungs.
  • To know which medication the doctor will prescribe.
  • To know if there is an affectation in the heart.

Further assessment is performed on Patient Mina, her history revealed that she is previously treated and is experiencing a relapse. What is her treatment regimen

  • 2HRZE/4HR
  • 2HRZE/10HR
  • 2HRZES/1HRZE/5HRE
  • 2HRZES/1HRZE/9HRE

After weeks of treatment Patient Mina went back to the hospital with the complaint of numbness, pain, and tingling in the hands and feet. Which drug may have caused this

  • Rifampin
  • Isoniazid
  • Pyrazinamide
  • Ethambutol

Nurse Grace is now performing health teaching. Which of the following she should not include

  • Encourage intake of tuna, aged cheese, red wine, and soy sauce.
  • Encourage intake of dark leafy vegetables, banana, and papaya.
  • Encourage adherence to treatment regimen.
  • Encourage socialization after 2 to 3 weeks of continuous medication therapy.

Which of the following would best indicate a maturational crisis

  • Illness
  • Death
  • Marriage
  • unemployment

Providing crisis intervention is an important part of:

  • Primary prevention
  • Health promotion
  • Secondary prevention
  • Tertiary prevention

Nurse Anne observes that the family is exhibiting strengths in the face of hardships they are facing. The best description of the strengths Nurse Anne is observing is:

  • Power
  • Resilience
  • Projection
  • Faith

Nurse Leslie is educating the family regarding the general need for adequate nutrition, rest, and physical activity. Which level of prevention does this describe

  • Tertiary prevention
  • Secondary prevention
  • Health maintenance
  • Primary Prevention

Nurse Alex is assigned to perform a Home visit to Brgy. Kasinay. Which of the following is the first step in conducting a home visit

  • Greet the patient and introduce self
  • State the purpose of the visit
  • Observe the patient and determine the health needs
  • Make an appointment for a return visit

As Nurse Alex is performing a home visit to family Lopez, they asked, “How often are you going to visit us?”. Nurse Alex shouldn’t respond

  • “It will be based on your physical, psychological needs, and educational needs”
  • “It will be based on your acceptance of the services to be rendered”
  • “It will be based on Nurse’s convenience”
  • “It will be based on the ability of the patient and his family to recognize their needs”

Nurse Alex then proceeded to perform the bag technique, which of the following is a principle of bag technique? Select all that apply.

  • Bag technique minimizes, if not prevents the spread of any infection
  • It saves time and effort in performance of nursing procedures
  • It should show effectiveness of total care given to family
  • It can be performed in a variety of ways depending on agency’s policy and home situation, as long as it avoids transfer of infection is always observed.

Situation

Rabies is endemic in the Philippines and remains to be a public health concern. Community Health Nurse Lisa provides health teaching regarding its prevention and control program.

Karen, a resident of the community, asked Nurse Lisa on the initial symptoms of rabies. Nurse Lisa is right when she answers:

  • “Karen, the initial symptoms of rabies include cough, body malaise, syncope, hyperemia and hematemesis”
  • “Karen, the initial symptom of Rabies include fever with pain and unusual or unexplained tingling, pricking or burning sensation at the wound site”
  • “ Karen, the initial symptom of rabies includes severe fatigue, muscle weakness, hyperthyroidism and painless wound site
  • “Karen, the initial symptoms of rabies includes burning sensation on wound site, hemorrhage, hematemesis, hematuria and hematochezia.”

Karen continues to ask Nurse Lisa, “How can rabies be transmitted?”

  • Highly contagious virus that lives in the nose and throat mucus of an infected person. It can spread to others through coughing and sneezing.
  • By being bitten by an infective female Anopheles mosquito.
  • Has lymphatic, microscopic worms circulating in the person’s blood enter and infect the mosquito.
  • When infectious material, usually saliva, comes into direct contact with a victim’s fresh skin lesions

The public health nurse is responsible for presenting the municipal health statistics using graphs and tables. To compare the frequency of the leading causes of mortality in the municipality, which graph will you prepare?

  • Line
  • Bar
  • Pie
  • Scatter diagram

Which step in community organizing involves training of potential leaders in the community?

  • Integration
  • Community organization
  • Community study
  • Core group formation

In which step are plans formulated for solving community problems?

  • Mobilization
  • Community organization
  • Follow-up/extension
  • Core group formation

Isolation of a child with measles belongs to what level of prevention?

  • Primary
  • Secondary
  • Intermediate
  • Tertiary

Which type of family-nurse contact will provide you with the best opportunity to observe family dynamics?

  • Clinic consultation
  • Group conferences
  • Home visit
  • Written communication

The youngest child of the Reyes family has been diagnosed as mentally retarded. According to categories of health problem, this is classified as:

  • Health Threat
  • Health Deficit
  • Foreseeable Crisis
  • Salience

The Miranda couple have 6-year old child entering school for the first time. The Miranda family has a:

  • Health Threat
  • Health Deficit
  • Foreseeable Crisis
  • Salience

Which of the following is an advantage of a home visit?

  • It allows the nurse to provide nursing care to a greater number of people
  • It provides an opportunity to do first hand appraisal of the home situation
  • It allows sharing of experience among people with similar health problems
  • It develops the family’s initiative in providing for health needs of its members

To maintain the cleanliness of the bag and its contents, which of the following must the nurse do?

  • Wash his/her hands before and after providing nursing care to the family members
  • In the care of family member’s, as much as possible, use only articles taken from the bag
  • Put on an apron to protect her uniform and fold it with the right side out before putting it back into the bag.
  • At the end of the visit, fold the lining on which the bag was placed, ensuring that the contaminated side is on the outside.

Which of the following is an epidemiologic function of the nurse during an epidemic?

  • Conducting assessment of suspected cases to detect the communicable diseases
  • Monitoring the condition of the cases affected by the communicable disease
  • Participating in the investigation to determine the source of epidemic
  • Teaching the community on preventive measures against the disease

Which law created the Philippine Institute of Traditional and Alternative Health Care?

  • RA 8423
  • RA 4823
  • RA 2483
  • RA 3482

Which of the following demonstrates inter-sectoral linkages?

  • Two-way referral system
  • Team approach
  • Endorsement done by a midwife to another midwife
  • Cooperation between PHN and public school teacher

Which statistic can give the most accurate reflection of the health status of a community?

  • 1-4 year old age-specific mortality rate
  • Infant mortality rate
  • Swaroop’s index
  • Crude death rate

This shows the relationship between a vital event and those persons exposed to the occurrence of said event

  • Ratio
  • Rate
  • Crude
  • Specific rate

The Field Health Services and Information System (FHSIS) is the recording and reporting system of public health care in the Philippines. The monthly field health service activity report is a form used in which of the components of the FHSIS?

  • Tally report
  • Output report
  • Target/client list
  • Individual health record

To monitor clients registered in long-term regimens, such as the Multi-Drug Therapy, which component will be most useful?

  • Tally report
  • Output report
  • Target/client list
  • Individual health record

This is the fundamental building block of the Field Health Services and Information System (FHSIS):

  • Tally report
  • Output report
  • Target/client list
  • Individual health record

Which of the following women should be considered as special targets for family planning?

  • Those who have two children or more
  • Those with medical conditions such as anemia
  • Those younger than 20 years and older than 35 years
  • Those who just had a delivery within the past 15 months

Freedom of choice in one of the policies of the Family Planning Program of the Philippines. Which of the following illustrates this principle

  • Information dissemination about the need for family planning
  • Support of research and development in family planning methods
  • Adequate information for couples regarding the different methods
  • Encouragement of couples to take family planning as a joint responsibility

Which immunization produces a permanent scar?

  • DPT
  • BCG
  • Measles vaccination
  • Hepatitis B vaccination

Inadequate intake by the pregnant woman of which vitamin may cause neural tube defects:

  • Niacin
  • Iron
  • Folic Acid
  • Thiamine

A 4-week-old baby was brought to the health center for his first immunization. Which can be given to him?

  • DPT1
  • OPV1
  • BCG
  • Hepatitis B Vaccine

You will not give DPT 2 if the mother says that the infant had

  • Seizures a day after DPT1
  • Fever for 3 days after DPT1
  • Abscess formation after DPT1
  • Local tenderness for 3 days after DPT1

A 2-month-old infant was brought to the health center for immunization. During assessment, the infant’s temperature registered at 38.1°C. Which is the best course of action that you will take

  • Continue with the infants immunization
  • Give paracetamol and wait for his fever to subside
  • Refer the infant to the physician for further assessment
  • Advise the infant’s mother to bring him back for immunization when he is well

A pregnant woman had just received her 4th dose of tetanus toxoid. Subsequently, her baby will have protection against tetanus for how long?

  • 1 year
  • 3 years
  • 10 years
  • Lifetime

Using IMCI guidelines, you classify a child as having severe pneumonia. What is the best management for the child?

  • Prescribe antibiotic
  • Refer him urgently to the hospital
  • Instruct the mother to increase fluid intake
  • Instruct the mother to continue breastfeeding

Which of the following signs will indicate that a young child is suffering from severe pneumonia?

  • Dyspnea
  • Wheezing
  • Fast breathing
  • Chest indrawing

A 5-month old infant was brought by his mother to the health center because of diarrhea occurring 4 to 5 times a day. His skin goes back slowly after a skin pinch and his eyes are sunken. Using the IMCI guidelines, you will classify this infant in which category?

  • No signs of dehydration
  • Some dehydration
  • Severe dehydration
  • Hypovolemia

According to the previous question, what would be your intervention based on the IMCI management guidelines?

  • Bring the infant to the nearest facility where IV fluids can be given
  • Supervise the mother in giving 200 to 400 ml of Oresol in 4 hours
  • Give the infant’s mother instructions on home management
  • Keep the infant in your health center for close observation

In IMCI, severe conditions generally require urgent referral to a hospital. Which of the following severe conditions does not always require urgent referral to a hospital?

  • Mastoiditis
  • Severe dehydration
  • Severe pneumonia
  • Severe febrile disease.

The pathognomonic sign of measles is Koplik’s spot. You may see Koplik’s spot by inspecting the:

  • Nasal Mucosa
  • Buccal mucosa
  • Skin on the abdomen
  • Skin on the antecubital surface

Among the following diseases, which may undergo airborne transmission?

  • Viral conjunctivitis
  • Acute poliomyelitis
  • Diphtheria
  • Measles

A 6-year-old client was brought to the health center with chief complaint of severe diarrhea and the passage of “rice water”. The client is most probably suffering from which condition?

  • Giardiasis
  • Cholera
  • Amebiasis
  • Dysentery

A 32-year-old man came for consultation at the health center with the chief complaint of fever for a week. Accompanying symptoms were muscle pains and body malaise. A week after the start of fever, the client noted yellowish discoloration of his sclera. History showed that he waded in flood waters about 2 weeks before the onset of symptoms. Based on this history, which disease condition will you suspect?

  • Hepatitis A
  • Hepatitis B
  • Tetanus
  • Leptospirosis

Sigmund Freud’s stages of psychosexual development, in chronological order, are

  • Oral, Anal, Phallic, Latent, and Genital
  • Oral, Anal, Latent, Phallic, and Genital
  • Oral, Genital, Anal, Phallic, and Latent
  • Oral, Phallic, Anal, Latent, and Genital

This acts immediately in an impulsive, irrational way and pays no attention to the consequences of its actions; therefore, often behaves in ways harmful to self and other.

  • Id
  • Ego
  • Superego
  • Conscience

This includes reality-testing and problem-solving and follows the reality principle:

  • Id
  • Ego
  • Superego
  • Conscience

This includes the internalization of the values, ideals, and moral standards of parents and society.

  • Id
  • Ego
  • Superego
  • Conscience

The clinic nurse is preparing to explain the concepts of Kohlberg’s theory of moral development with a parent. The nurse should tell the parent that which factor motivates good and bad actions for the child at the preconventional level?

  • Peer pressure
  • Social pressure
  • Parents’ behavior
  • Punishment and reward

A client with a diagnosis of depression who has attempted suicide says to the nurse, “I should have died. I’ve always been a failure. Nothing ever goes right for me.” Which response by the nurse demonstrates therapeutic communication?

  • “You have everything to live for.”
  • “Why do you see yourself as a failure?”
  • “Feeling like this is all part of being depressed.”
  • “You’ve been feeling like a failure for a while?”

The nurse visits a client at home. The client states, “I haven’t slept at all the last couple of nights.” Which response by the nurse demonstrates therapeutic communication?

  • “I see.”
  • “Really?”
  • “You’re having difficulty sleeping?”
  • “Sometimes I have trouble sleeping too.”

A client diagnosed with terminal cancer says to the nurse, “I’m going to die, and I wish my family would stop hoping for a cure! I get so angry when they carry on like this. After all, I’m the one who’s dying.” Which response by the nurse is therapeutic?

  • “Have you shared your feelings with your family?”
  • “I think we should talk more about your anger with your family.”
  • “You’re feeling angry that your family continues to hope for you to be cured?”
  • “You are probably very depressed, which is understandable with such a diagnosis.”

The nurse employed in a mental health clinic is greeted by a neighbor in a local grocery store. The neighbor says to the nurse, “How is Carol doing? She is my best friend and is seen at your clinic every week.” Which is the most appropriate nursing response?

  • “I cannot discuss any client situation with you.”
  • “If you want to know about Carol, you need to ask her yourself.”
  • “Only because you’re worried about a friend, I’ll tell you that she is improving.”
  • “Being her friend, you know she is having a difficult time and deserves her privacy.”

A client says to the nurse, “The federal guards were sent to kill me.” Which is the best response by the nurse to the client’s concern?

  • “I don’t believe this is true.”
  • “The guards are not out to kill you.”
  • “Do you feel afraid that people are trying to hurt you?”
  • “What makes you think the guards were sent to hurt you?”

A client is unwilling to go to his church because his ex-girlfriend goes there and he feels that she will laugh at him if she sees him. Because of this hypersensitivity to a reaction from her, the client remains homebound. The home care nurse develops a plan of care that addresses which personality disorder?

  • Avoidant
  • Borderline
  • Schizotypal
  • Obsessive-compulsive

The nurse is developing a teaching plan for a client with glaucoma. Which instruction should the nurse include in the plan of care?

  • Avoid overuse of the eyes.
  • Decrease the amount of salt in the diet.
  • Eye medications will need to be administered for life.
  • Decrease fluid intake to control the intraocular pressure.

The nurse is performing an admission assessment on a client with a diagnosis of detached retina. Which sign or symptom is associated with this eye problem?

  • Total loss of vision
  • Pain in the affected eye
  • A yellow discoloration of the sclera
  • A sense of a curtain falling across the field of vision

A client is diagnosed with a problem involving the inner ear. Which is the most common client complaint associated with a problem involving this part of the ear?

  • Pruritus
  • Tinnitus
  • Hearing loss
  • Burning in the ear

Situation

Nursing theories play a vital role in the evolution of nursing as a discipline. They not only establish the critical contributions made by the profession, but also provide nurses with frameworks to guide their practice, grow professionally, and improve the care they deliver to patients. First year student Nurse Angela is assigned to make a report about nursing theorists.

The four major concepts in nursing theory are:

  • Person, Environment, Nurse, Health
  • Nurse, Person, Environment, Cure
  • Promotive, Preventive, Curative, Rehabilitative
  • Person, Environment, Nursing, Health

The act of utilizing the environment of the patient to assist him in his recovery is theorized by:

  • Nightingale
  • Benner
  • Henderson
  • Levin

She believe the unique function of the nurse is to assist the individual, sick or well, in the performance of those activities contributing to health that he would perform unaided if he has the necessary strength, will and knowledge, and do this in such a way as to help him gain independence as rapidly as possible.

  • Henderson
  • Abdellah
  • Nightingale
  • Peplau

Nurse Sabrina is planning a seminar on leadership styles. Which of the following statements describes a democratic leadership style?

  • The leader assumes a “hands-off” approach.
  • Under this leadership style, the group may feel secure because procedures are well defined and activities are predictable.
  • This leadership style demands that the leader have faith in the group members to accomplish the goals.
  • This leadership style does not trust self or others to make decisions and instead relies rules, policies, and procedures to direct the group’s work efforts on the organization’srules, policies, and procedures to direct the group’s work efforts

A Chief Nurse who fosters creativity, risk taking, commitment, and collaboration by empowering the group to share in the organization’s vision is which type of leader?

  • Charismatic
  • Transactional
  • Transformational
  • Shared

The head nurse who has the ability and willingness to assume responsibility for one’s actions and to accept the consequences of one’s behavior is demonstrating what management principle?

  • Accountability
  • Authority
  • Responsibility
  • Coordinating

Situation

Nurse Danielle is planning a seminar on wellness care and promoting fluid and electrolyte balance.

All except one action is appropriate for Nurse Danielle who is starting an intravenous (IV) infusion?

  • Adjusts the IV pole so that the solution container is suspended about 1 meter (3 feet) above the client’s head.
  • Completely fills the drip chamber with solution.
  • Uses the client’s nondominant arm, unless contraindicated.
  • Cleans the skin at the site of entry with a topical antiseptic swab

As Nurse Danielle is doing her rounds one patient caught her attention. The patient has complaints of discomfort in her IV site. Nurse Danielle assesses the patient and finds localized swelling, coolness, and pallor. The patient is experiencing:

  • Extravasation
  • Phlebitis
  • Infiltration
  • Hypersensitivity reaction

A client complains of a headache 10 minutes after the transfusion of a unit of packed red blood cells was initiated. The nurse assesses that the client has slight shortness of breath and feels warm to the touch. What action by the nurse is priority

  • Notify the client’s physician.
  • Discontinue the transfusion.
  • Slow the rate of the transfusion.
  • Prepare to resuscitate the client.

A client was discharged after having a 1-day surgery on her gallbladder. The registered nurse discharging the client failed to give the client oral or written discharge instructions. This failure to carry out the provision of discharge instructions could result in charges of?

  • Malpractice
  • Negligence
  • Assault
  • Battery

A nurse threatens to give a loud, disruptive client an injection that will “knock the client out.” The nurse follows through on the threat and gives the injection Without the client’s consent. What has the nurse committed?

  • Threat, Assault
  • Battery, Invasion of privacy
  • Assault, Invasion of privacy
  • Assault, Battery

Nurse Richard is taking care of an adult client when he throws a temper tantrum because he does not get his own way. Which defense mechanism is the adult client displaying?

  • Repression
  • Regression
  • Reaction formation
  • Rationalization

A client comes into the clinic with tremors and pitch changes in her voice. She also has facial twitches and shakiness. Her respiratory and heart rates are slightly elevated. At the end of her assessment she tells you, “I feel like I have butterflies in my stomach.” Which level of anxiety is this client experiencing?

  • Mild
  • Moderate
  • Severe
  • Panic

Recalls IV

Situation

The Family Nursing Care Plan is designed to provide ways in solving health-related problems of the family as a whole. The nursing process is used to make this type of care plan.

In conducting a family assessment, which source of data would be most helpful?

  • Input from other members of the health team.
  • Information from other cooperating health care agencies.
  • Review of family members’ charts and medical record data.
  • Observation and interaction with the family members.

In completing a family assessment, the public health nurse should begin by:

  • Gathering the health data from all family members
  • Testing the family’s ability to cope
  • Evaluating communication patterns
  • Identification of the geographic location of resources for the family.

Which of the following would be better understood by the nurse after using a family map?

  • Roles and relationship within the family
  • Recognition of the leade who speaks for the family.
  • Description of the social and economic status of the family.
  • Identification of the geographic location of resources for the family.

The nurse discusses with the family some actions that needed to be taken and refers to the wife in the family as an outstanding organizer. Which of the following goals is the nurse trying to achieve by making this statement?

  • Making then husband proud that he had married his wife.
  • Reinforcing for the family some of the strengths of one of their family members.
  • Encouraging the wife to volunteer to follow through on all the needed actions.
  • Making the wife feel very complimented and proud.

In implementing family-centered care, the public health nurse:

  • Works with clients to help them accept limitations for their actions.
  • Assists family members to assume dependent roles
  • Offers information about necessary self-care abilities.
  • Provides his or her own beliefs on how to solve problems

Public Health Nurse Silang is educating a group of residents in Community Marahas about rabies. A participant asks, “How can you get the virus?” The Nurse explains that the virus can be transmitted by which source?

  • Saliva
  • Blood
  • Urine
  • Stool

She emphasized that the bites, which are responsible for nearly 99% of human rabies infections, are those of the infected:

  • Monkeys
  • Dogs
  • Bats
  • Cats

Nurse Silang explains that there are situations which increase the risk of rabies. These are:

  • Stray of dogs
  • All of these
  • Unprovoked bites
  • Animals display abnormal behavior

In another health education session, Nurse Silang explains about leprosy. Which of the following should be included? Select all that apply.

  • Leprosy is curable
  • Not all leprosy patients are infectious
  • Leprosy is hereditary
  • Casual contact with a patient causes leprosy
  • Regular and adequate treatment is essential

Nurse Silang stated that a definitive diagnosis of leprosy is obtained through?

  • Blood examinations
  • Tuberculin testing
  • Skin smears/Biopsies
  • Nasal smears

Situation

Nurse Nikito is assigned in a disaster- prone province in the Visayas. He is aware that with increase frequency of disaster happening, he has to respond quickly and efficiently to assist the population affected by calamities.

Which of the following best defines a disaster?

  • Any event that results in multiple deaths.
  • Devastation that covers a broad geographic area.
  • Devastation that cannot be relieved without assistance.
  • The event results in multiple injuries, deaths and property damage

Nurse Nikito is guided by the ICN framework of disaster nursing competencies. This framework consists of four areas in the continuum of disaster management that corresponds to the four stages of disaster. What consists of the first stage?

  • Response
  • Preparedness
  • Prevention
  • Recovery

Nurse Nikito has invited several agencies in the community to a meeting to discuss the disaster plan for the community. Which of the following best describes the purpose of this meeting?

  • Enhance communication among agencies in the community
  • Increase stability in the community
  • Manage response to disasters in the community
  • Improve overall community functioning

Nurse Nikito adapts professional nursing skills in recognizing and meeting the physical and emotional needs resulting from a disaster. For people who are willing to talk ensuing a disaster, which of the following in the most appropriate approach?

  • “I am with you. It is good you are trying to release your distress by crying. It will make you feel better. “
  • “What you need to do now is to wait for instructions and services to be provided.”
  • “Don’t feel bad. Others are in the same situation as yours.”
  • “You need not cry. You need to move on and build your life again.”

As he passed by a road going to an evacuation center, Nurse Nikito encountered a flash flood. A flash flood:

  • Occurs suddenly and for a short duration
  • Is caused by the blocking of drains
  • Is caused by heavy rains
  • Occurs in urban areas

Situation

Albert is a 15-year-old adolescent who lives with his mother father, and several siblings. He is hospitalized in a provincial hospital after a suicide attempt.

When performing a family assessment, select what the nurse must first determine.

  • How the family expresses and manages emotions
  • The communication patterns between the patient and parents
  • Names and relationships among family members
  • The meaning that the patient’s suicide attempt has for family members

Which of the following is the best question for the nurse to ask to assess a family’s ability to cope?

  • “Do you think your family copes effectively?”
  • “What do you think of the current family problem?”
  • “What strengths does your family have?”
  • “Can you describe how you successfully handled one family problem?”

The mother of Albert asks the nurse, “Why do you want to do a family assessment? My teenager is the patient, not the rest of us.” What is the best response of the nurse?

  • “Family dysfunction might have caused the mental illness.”
  • “Every family member ‘s perception of events is different and adds to the total picture.”
  • “Family members provide more accurate information than the patient.”
  • “Family assessment is a protocol for care of all patients with mental illness.”

Which information is the nurse most likely to find when assessing the family of a patient with mental illness?

  • Power in the family is maintained in the parental dyad and rarely delegated.
  • Several family members have serious problems with their psychological health.
  • Stress that living with a mentally ill member has challenged the family’s function.
  • The family exhibits many characteristics of dysfunctional families.

Parents of Albert say, “We have never known anymore who was mentally ill. We have no one to talk to because none of our friends understand the problems we are facing.” Which of the following is the nurse’s most helpful action?

  • Facilitate achievement of normal developmental tasks of the family.
  • Build the parent’s self-concept as coping patterns.
  • Teach the parents techniques of therapeutic communication.
  • Refers the parents to a support group.

Situation

In public health service, good record system is important in the delivery of quality care to clients. As a public health nurse, you have the responsibility of assisting in the preparation, maintenance of good records and reports.

You want to distinguish between records and reports. As applied to public health, the following are types of records, except:

  • Events such as births, illness, deaths
  • Analysis of a problem situation
  • Clinic consultation
  • Written data on home visit

Reports are prepared usually for administrative purposes. Which of the examples below is not a report?

  • Summary of services delivered
  • Accomplishments and failures of services
  • Description of a program and planned services
  • Client information and profile

There are commonly used record and reports in public health nursing practice. The individual clinical record contains which of the following information? Select all that apply.

  • Socio-demographic characteristics
  • Patient’s chief complaint
  • Physical Examination findings
  • Prescription of drugs
  • Frequency of clinic check up
  • Attendance to community settings

The family Service and Progress Record (FSPR) serves as a tool to operationalize the concept of the family as the unit of care. The FSPR consists of four parts, the first being?

  • Health and nursing problems
  • Nursing care plan
  • Assessment of the family and the environment
  • Service and Progress Notes

Which of the following relates to the family’s nursing problem?

  • Inability to assume health tasks with respect to health problems
  • Crisis situation or developmental deficite
  • Health threats, risks or hazards
  • Strained relationships or disunity

Situation

Ms. Mahilom, a public Health Nurse Supervisor, is a preparing a supervisory plan for midwives under her charge. A supervisory plan is a written document on how to organize and systematizes supervisory activities.

The need for supervision may arise from which of the following? Select all that apply

  • Lack of motivation
  • Conflict between personal and organizationalgoals.
  • Lack of knowledge and skills
  • Desire for promotion or job permanency
  • Achieve health goals for the agency

Ms. Mahilom would require information regarding the supervisory needs of the midwives which can be taken from which of the following?

  • Review of records and reports
  • Interview of the midwives
  • Review of literature
  • Observation of the midwives at work
  • Results of survey among the staff.

Ms. Mahilom prioritizes supervisory needs and problems based on the following criteria, except?

  • Convenience to both parties to address needs
  • Degree of importance of the identified need
  • Availability of resources needed
  • Activities needed to meet the identified need

During the actual supervisory visit, which of the following is not expected of Ms. Mahilom? She:

  • Discusses the objective of the visit
  • Expresses appreciation and support given for the visit
  • Conveys a formal, strict approach to the midwives
  • Explains the process and outcomes of the visit

After setting the objectives and selecting the activities, Ms. Mahilom would need to identify the indicators for evaluation. Which one is not included?

  • Needs met
  • Performance increased
  • Promotion achieved
  • Quality of service improved

Situation

As a public health nurse, Ms. Sisa intends to focus on a specific population to advocate, educate, collaborate with members from a community to improve health of the people.

Nurse Sisa collects data and monitors the health status of the population. Which of the following core public functions is being implemented?

  • Assurance
  • Policy development
  • Assessment
  • Prevention

Nurse Sisa included in her data collection the number and proportion of persons aged 25 or older with less than a high school education. Which of the following best describes this data?

  • Health status data
  • Health care resource information
  • Health risk factors
  • Sociodemographic data

Working in the community with an aggregate/population, who does Nurse Sisa most likely to interact with?

  • Students in a local high school
  • Patients at the local hospital
  • Residents who play basketball
  • Christians in the community

Nurse Sisa has a clear understanding of population-focused practice. Which of the following characteristics would she most likely display?

  • Improving the effectiveness of care provided
  • Sponsoring a fund-raising project
  • Volunteering for a community action
  • Providing health interventions for individuals

Nurse Sisa is working to improve population-focused care in the community. Which of the following best described a key opportunity for nurses like her to accomplish this goal?

  • Assuming traditional nursing roles
  • Conducting community assessments
  • Specializing in community health practice
  • Influencing public health policy

Situation

Ethical issues on a day-to-day basis often involve patient communication and advocacy. It may also questions about end-of-life care. Nurses are often the one working closet with patients and families while administering care.

What is the appropriate approach for the community/public health nurse in balancing individual privacy and community’s needs for safety and security?

  • Fulfill agency policies first.
  • Implement professional standards of nursing practice.
  • Remember the group needs outrank individual needs.
  • Seek a balance between individual’s need and the community’s needs.

In regard to informed consent, which of the following statements is true?

  • Client may not be told about costs and alternatives to treatment.
  • Consent must be voluntary.
  • Only parents themselves should give consent for minor children.
  • Legally incompetent adults can give consent.

The client has a living will in which he states he does not want to be kept alive by artificial means even at home. The client’s family wants to disregard the client’s wishes and have him maintained on artificial life support. The most appropriate initial course of action for the nurse would be to:

  • Allow the family to ventilate their feelings and concerns, while maintaining the role of client advocate.
  • Tell the family that they have no legal rights.
  • Report the situation to the hospital’s Ethics Committee.
  • Advise the family that they have the right to ignore the living will as the patient cannot speak.

During a routine visit, the nurse noted that Perlita, 20-year old wife and mother, has several bruises at various stages of healing. She tells the nurse that she fell down. Failure to report your findings is an example of?

  • Malpractice
  • Reasonable Prudence
  • Maleficence
  • Negligence

Which of the following examples would indicate observance of client confidentiality?

  • Reading a friend’s chart in another health care facility.
  • Describing a difficulty with a client in a health team conference.
  • Using a client’s name in a social conversation.
  • Talking about the client’s symptoms in front of family members.

Situation

Non-communicable Diseases (NCDs) remain to be the major health challenges in the Philippines and globally. The specific population group that are mostly afflicted by these chronic conditions, such as diabetes and cardiovascular diseases, are those aged 60 and above.

When caring for a patient with type 2 diabetes who has been discharged, which topic will be most important to include in your health teaching?

  • Impact of the patient’s family history on likelihood of developing diabetes.
  • Symptoms indicating that the patient should contact the health care provider.
  • Effect of endogenous insulin on transportation of glucose into cells
  • Function of the liver in formation of glycogen and gluconeogenesis

To obtain the most complete information when doing an assessment for a 75-year-old patient, you will:

  • Ask the patient to write down medical problems and medications
  • Use a geriatric assessment instrument to evaluate the patient.
  • Interview both the patient and the primary patient caregiver
  • Review the patient’s chart for the history of medical problems

Which information about a 77-year-old patient who is being assessed by the public health nurse us of utmost concern? The patient:

  • Says, “I don’t go on my daily walks since I had pneumonia two months ago.”
  • Tells the nurse, “I prefer to manage my life without much help from others.”
  • Uses three different medications for chronic heart and joint problems.
  • Organizes medications in a marked pillbox “so I don’t forget them.”

When caring for an older adult who lives in a rural area, you will:

  • Ensure transportation to appointments with the health care provider.
  • Assess the patient for chronic diseases that are unique to rural areas.
  • Obtain adequate medications for the patient to last for 4 to 6 months.
  • Suggest that the patient move to an urban area for better health care

Which of these patients assigned to you is most likely to need planning for long-term nursing management?

  • 71 year-old with appendicitis who has had an emergency appendectomy
  • 60 year-old with bilateral knee osteoarthritis who weighs 350 lbs (159 kg)
  • 54 year-old with cholecystitis who has had a laparoscopic cholecystectomy
  • 62 year-old with acute sinusitis who will require antibiotic therapy for 5 days

Situation

Assessment at the community-level of care is a multidisciplinary undertaking that involves the member of the health team. The public health nurse uses data that have already been collected and are available and or gathers primary data for community diagnosis.

Assessment is the first step in the nursing process. In the setting, which of the following objectives of assessment by the public health nurse is not included?

  • Identify specific risk factors related to health and health problems.
  • Define the nature of the health status and health-related problems
  • Identify clients who should be given priority for care.
  • Determine who should be referred to different health care facilities.

Conducting a community assessment leads to a community diagnosis, its products consists of a profile of the community’s state of health. As a process, the public health nurse actively participates in community diagnosis. The most common method of data collection that is accurate and provides the biggest bulk of community data is the:

  • Observation
  • Community census
  • Interview
  • Records review

The data on health and disease can be gathered from different sources. Which of the following sources provide a most accurate set of data?

  • Population health profile from school and factory clinics.
  • Births and death from city/municipality registrar.
  • Morbidity data from filed personnel.
  • Mortality cases from medical death certificates.

Data analysis involves qualification, description and classification of data which would reveal community health problems. An analysis of the social, economic, environmental and political factors that influence health is categorized as:

  • Health resources problems
  • Health status problems
  • Health-related problems
  • Comprehensive problems

Situation

Collaboration is being able to work cooperatively and efficient with co-workers and any other people the nurse may have to interact with. This includes other health professionals, clients, and their families and communities

With the stated problems, the nurse and the community prioritize these using a set of criteria. Which of the following criteria refers to the probability of reducing, controlling or eradicating the problem?

  • Modifiability of the problem
  • Preventive potential
  • Magnitude of the problem
  • Nature of the problem

Situation

Professional and personal developments are required in order to maintain and enhance professional standards and to provide quality, competent and safe patient care.

Identify the element that is the BEST indicator of increasing accountability in the profession of Nursing:

  • Increasing salary scale for nurses.
  • Improved public image of nurses in social media
  • Increasing number of nurses with doctoral degree
  • Demonstrate competency and high-quality care

Which of the following is the most important element in nursing’s attempt to gain full autonomy of practices?

  • Maintaining the education system for nurses as it is now
  • Economic well-being and welfare of nurses
  • Restricting the latitude of decisions made by nurses
  • Gaining and maintaining control of nursing practice by nurses

What is the best method for nurses to prepare for future professional practice?

  • Train other health care workers to whom delegation of tasks can be given.
  • Understand and explore the issues involved in professional practice
  • Establish a coalition with professional health organizations
  • Take additional courses in the use of computers and telemetry

What allows a nurse to exert legitimate power over a client when providing nursing care?

  • The ability to perform procedures to alleviate patient’s discomfort
  • The ability to provide the client and family incentives and for self care
  • The power given to the nurse by reason of licensure examination and registration
  • The establishment of a professional and personal relationship with the client

A nurse is employed by a health care agency that provides an informal training session on how to properly use a new vital sign monitor. Which type of education did the nurse receive?

  • Career counseling
  • Continuing education
  • Pre-employment education
  • In-service education

Situation

The Public Health Foundation, an NGO, defines quality improvement in public health as “the use of a deliberate and defined improvement process, such as the PDCA which is focused on activities that are responsive to community needs and improving population health.” Public Health Nurse Rosita learned this management process during her undergraduate studies in nursing.

The acronym PDCA stands for:

  • Plan-Do-Check-Act
  • Program-Design-Check-Act
  • Plan-Draft-Certify-Act
  • Prepare-Do-Check-Achieve

Public Health Nurse Rosita will use the PDCA when doing which of the following? Select all that apply.

  • Working towards continuous improvement
  • Implementing any change
  • Developing a new design of a process or service
  • Completing a designed project
  • Planning data collection and analysis to prioritize problems

Using the PDCA cycle as a model for defining most of the work processes the following are some areas of its applications, except:

  • Needs analysis
  • Overall strategic planning
  • Staff goal setting and evaluation
  • Delegation of work to lower levels

Public Health Nurse Rosita knows that the purpose of continuous quality improvement (CQI) is to:

  • Provide opportunity for staff members to participate in self-governance
  • Restructure health care delivery to improve cost savings
  • Deliver appropriate nursing care at tertiary level of service
  • Monitor processes involved in the provision of safe, effective client care

Public Health Nurse Rosita is implementing quality performance standards in a public health department. Which of the following BEST describes the importance of this action? Quality performance standards:

  • Can be used as hiring guidelines for nurses
  • Are used to guide improvement in the public health system
  • Guide administrators to monitor public health at the national level
  • Rigidly control public health

Situation

The World Health Organization stated that occupational health services, provided at the workplace to address the health care needs of working populations, have been identified as an important component of the public health strategy.

Occupational health services can make a significant contribution to government initiative. These include the following, reducing:

  • Health insurance
  • Health inequalities
  • Social exclusion
  • Sickness absence
  • Overall burden of ill health

The nurse is a key figure involved in delivering quality occupational health services. They work independently or as part of a larger interprofessional health team. They perform, along with others, a job safety analysis. In doing so, the following methods may be employed, except:

  • Review of records, interviews, surveys
  • Walk-through, process and output reviews
  • Observation, focused group discussion
  • Racial background check

The head of the health team creates a workplace safety report for a new clinic. He makes a strict recommendation to ensure that sharps are properly disposed in a puncture-proof container to decrease the risk of needle-prick injuries. This upholds safety under which of the following categories?

  • Physical
  • Chemical
  • Biological infections
  • Psychosocial

The occupational nurse has advocated the use of ramps and assistive equipment for physically challenged workers. This is good example of which of the following levels of prevention?

  • Health promotion
  • Secondary
  • Tertiary
  • Primary

Every employee has right to privacy and should be protected from unauthorized and inappropriate disclosure of personal information. Which situation does not provide exception to employee’s right to privacy?

  • Authorization by the employee to release information
  • Life-threating emergencies
  • Compliance with government laws and regulations
  • Request from community leaders

Situation

Public Health Nurse Alma is a member of the health Team that is tasked by the Rural Health Unit to identify, define and measure major health problems of interest.

Public health surveillance includes the following activities, except:

  • Data analysis
  • Data collection
  • Disease control
  • Data interpretation

Public health surveillance can be best described primarily as which of the following:

  • A system for collecting health-related information
  • A method to monitor occurrences of public health problems
  • A program to control disease outbreaks
  • A system for monitoring persons who have been exposed to a communicable disease

Which of the following is not a criterion for prioritizing health problems for surveillance?

  • Incidence of the problem
  • Number of previous studies of the problem
  • Social and economic impact of the problem
  • Public concern about the problem

Current public health surveillance targets the following. Which one is not included?

  • Chronic disease
  • Population migration
  • Communicable disease
  • Occupation Hazard

Common uses and application of public health surveillance include which of the following examples?

  • Detecting individual persons with malaria so that they can receive prompt and appropriate treatment.
  • Helping public health officials decide how to allocate their disease control resource.
  • Identifying changes over time in the proportion of children with elevated blood levels in a community.
  • Documenting changes in varicella (chicken pox) incidence, after a law mandating varicella vaccination in the Expanded Program of Immunization took effect.

Situation

You are a newly-assigned Rural Health Nurse and you have the based on the latest health information of the municipality, food- and water-transmitted parasitic infections are prevalent.

The factors that contribute to the occurrence of food-transmitted diseases are the following, except:

  • Consumption of improperly cooked food
  • Eating habits of the population such as eating raw food
  • Lack of fuel for cooking food
  • Overconsumption of locally available food

Paragonimus westermani or the oriental lung fluke causes in the human population by eating inadequately cooked crabs. Patients with paragonimiasis are usually misdiagnosed to have:

  • Malaria
  • Bacillary dysentery
  • Typhoid fever
  • Pulmonary tuberculosis

The eggs of Taenia solium or taenia saginata may be ingested by the pigs or cattle who serve as intermediate hosts. Which of the following measures will prevent the access of animals from ingesting Taenia eggs?

  • Quarantine or isolate the animals Foods
  • Feeding the pigs and cattle with clean foods.
  • Proper disposal of human faces
  • Protect the animals by giving them vitamins.

Prevention of Infection from Entamoeba histolytica is best done through which of the following practices?

  • Eating only organic vegetables
  • Washing food very well
  • Cooking food properly
  • Boiling of water from questionable source

Giardiasis is a disease caused by a flagellated protozoan and its prevalence is associated with which of the following conditions? Select all that apply.

  • Poor environmental sanitation
  • Poor hygiene
  • Improper cooking of food
  • Overcrowding
  • Malnutrition

Situation

Public Health Nurse Tess wants to increase her knowledge a familiarity with the elements of a research publication.

Public health Nurse Tess is analyzing a research article. What section in the article can she expect to find the research question and study purpose?

  • Discussion
  • Results
  • Methods
  • Introduction

Public Health Nurse Tess is looking for a description of the type of measurement used in the study. She will find this which section of the article?

  • Conclusion
  • Introduction
  • Results
  • Methods

A brief explanation of data collection and analysis procedure is found in which section of the article?

  • Abstract
  • Conclusion
  • Discussion
  • Introduction

Public Health Nurse Tess wants to go over the list of references used in the study which can be found:

  • At the end of the article
  • In the results section
  • As part of the literature review
  • In the introduction

Public Health Nurse Tess believes that research is significant to the nursing profession, hence, the study results should be disseminated. This is because research allows:

  • A specialized body of knowledge to be generated for use in health care delivery.
  • The scope of nursing practice to be expanded into areas formerly reserved for other disciplines.
  • Nursing responsibility to be more specifically defined
  • Liability within the practice of nursing to be decreased

Situation

Nurses perform duties that require physical assessments, critic thinking, and collaborative treatment plans, to name a few. Thus, nurses must understand the laws that affect their practice, as well as their legal responsibilities in their professional roles.

A nurse must follow laws that protect public health, safety, and welfare. Which law is the nurse following?

  • Code of Ethics
  • Standards of Practice
  • Continuing Professional Development
  • Nursing Practice Act

Which situation supports the charge of malpractice against a professional nurse?

  • A failure on the part of the nurse to establish a therapeutic relationship with the patient.
  • A failure on the part of the nurse to ensure that patients only receive care for which they could pay.
  • A failure on then part of then nurse to exercise reasonable and prudent care in treating a patient.
  • A failure on the part of the nurse to allay a patient’s fears about an upcoming procedure.

What does the court consider in determining the nurse’s liability for standards of care?

  • Professional education, experience, and specific conduct.
  • Professional experience, but not education or conduct.
  • Professional conduct, but not experience or education
  • Professional education, but not experience or conduct

The board of nursing has brought action against a nurse’s license based upon violation of a regulation. What is true about this scenario?

  • Rules and Regulations are internal to the Board, not the nurse.
  • Rules and regulations are only suggested standards or care and do not have to be followed.
  • These rules and regulations have the force of law.
  • Violation of a rule and regulation is not the same as violation of the nursing Practice Act.

The patient, injured at work, was seen by the factory occupational nurse. The nurse treated the wound and instructed the patient to get a tetanus antitoxin injection at the City Health Center. The patient failed to follow instructions, developed tetanus, and subsequently filed a suit against the nurse. What is the most likely result of the ensuing trial? The nurse is:

  • Liable, because there was no follow-up to ensure that the patient receive the injection
  • Not liable for damages, because the nurse has a right to expect that instructions will be followed.
  • Liable, because tetanus is easily treatable after diagnosis
  • Not liable, because tetanus is a reportable disease and the health center should have insisted ten treatment.

Situation

The ability to communicate is a very important skill for every nurse to have. The nurse must do so in an effective, caring and professional manner, especially when communicating with patients and their families.

A nurse wants to present information about influenza immunization to the older adults in the community. Which type of communication should the nurse use?

  • Signified meaning is wrong
  • Affect is appropriate
  • Personal space was violated
  • Vocabulary is poor

A nurse is standing beside then patient’s bed. Nurse: “how are you doing?” Patient: “I don’t feel good.” Which of the following communication elements is categorized as feedback?

  • “How are you doing?”
  • “I don’t feel good.”
  • Nurse is standing
  • Patient in bed

A smiling patient angrily states, “I will not cough and deep breathe.” How will the nurse interpret the finding? The patient’s:

  • Signified meaning is wrong
  • Affect is inappropriate
  • Personal space was violated
  • Vocabulary is poor

Before meeting the patient, a nurse talks to other caregivers about the patient. Which phase of the helping relationship is the nurse in with this patient?

  • Termination
  • Pre-interaction
  • Working
  • Orientation

During the initial home visit, the nurse lets the patient know that the visits are expected to end in about a month. Which phase of the helping relationship is the nurse in with this patient?

  • Working
  • Pre-interaction
  • Orientation
  • Termination

Situation

After passing the Nurses Licensure Examination, Nurse Tawi-Tawi takes her first job in a Rural Health Unit in Municipality Z. She reviews concepts and principles in Primary Health Care (PHC).

The key global agency that initiated the Alma Ata Conference on Primary Health Care is the:

  • World Bank
  • World Health Organization (WHO)
  • CARE International
  • United Nations Children’s Fund (UNICEF)

Which Asian country is considered among the first to adopt the principle of PHC?

  • Malaysia
  • Indonesia
  • Thailand
  • Philippines

Nurse Tawi-Tawi is aware that she has to use “tools and methods that are suitable and acceptable to the families and communities” that she serve. This PHC principle is referred to as:

  • Cultural application
  • Appropriate application
  • essential methodology
  • community participation

Nurse Tawi-Tawi recognizes that health services offered are to in accordance to the prevailing beliefs and practices of the intended clients of care. This PHC principle is known as:

  • Availability
  • Acceptability
  • Affordability
  • Accessibility

Collaboration and cooperation is crucial to the achievement of PHC goals. The referral system among the RHU, non-government organization, and local social welfare and development office is classified as what type of relationship?

  • Intra-sectoral collaboration
  • Coalition building
  • Inter-sectoral collaboration
  • Network linkage

Which of the following is not recognized as part of the nurse’s role as member of the health team?

  • Care manager and supervisor
  • Provider of care and education
  • Patient advocate and supporter
  • Performing physician’s functions with supervision

The health team has brainstormed for ideas on creating a more collaborative environment in a health care facility. Which idea or action would be the best to develop further?

  • Develop a plan for the different team members to follow and show impact of their work.
  • Emphasize that no one profession has all the necessary competence to provide all care.
  • Hold education sessions regarding collaboration for each unit.
  • Institute collaborative strategies that are traditionally practiced

Which patient scenario describes the best example of professional collaboration?

  • The nurse, physician, and physical therapist have all visited separately with the patient.
  • The nurse mentions to the physical therapist that the patient may benefit from muscle strengthening evaluation
  • The nurse and physician discuss the patient’s muscle weakness and initiate a referral for physical therapy.
  • The nurse, physical therapist, and physician have all developed separate care plans for the patient.

Which activities are appropriate for the nurse to collaborate with a patient? Select all that apply.

  • Prescribing a new medication dose
  • Health Promotion activities
  • End-of-life comfort decisions
  • Interpreting laboratory results
  • Lifestyle changes to improve health

The Rural Health Nurse and the Rural Health Physician are explaining the home care that will be needed by a patient with Tuberculosis. The patient’s spouse states angrily that it will not be possible to provide the care recommended. What is the best response by the nurse?

  • “I would like to listen to your concerns about the prescribed care.”
  • “It is important that you do. What the physician has prescribed.”
  • I can come back after you talk with your spouse about the care.”
  • “Let me review what is needed again, then I’ll talk to you later.”